Forum: Mechanik, Gehäuse, Werkzeug Magnetmotor Idee


von Steven (muellermilch)


Angehängte Dateien:

Lesenswert?

Hallo zusammen,

für meine Idee möchte ich das Magnetfeld eines Magneten in 2 Hälften 
teilen.
Einmal in die Beschleunigungsseite und einmal in die Bremsseite. (siehe 
Bild "TeilungMagnetfeld")
Um einen Magnetmotor bauen zu können, ist die Hauptbedingung ein 
Ungleichgewicht im Magnetfeld zu finden bzw. zwischen der 
Beschleunigungsseite und der Bremsseite. Wenn ich es nun schaffe, die 
Beschleunigungsseite größer bzw. stärker als die Bremsseite zu bauen, 
habe ich ein Ungleichgewicht. Dieses Ungleichgewicht muss nun größer 
sein, als der Reibungsverlust der gesamten Strecke und muss zusätzlich 
soviel Überschuss übrig haben, um die Bremsseite zu verlassen. Sind 
diese Bedingungen erfüllt, darf ich von einem Perpetuum Mobile reden.
Nun bin ich der festen Überzeugung mit meiner Idee ein Ungleichgewicht 
geschaffen zu haben. Meine Beschleunigungsseite ist also größer als 
meine Bremsseite. Ob allerdings nun dieses Ungleichgewicht ausreicht, 
sowohl den Reibungsverlust auszugleichen und genügend Überschuss übrig 
zu haben, gilt es zu klären und herauszufinden.

Mein Magnetmotor soll grob so aussehen: siehe Gif "mm"
Natürlich ist das noch kein funktionierender Magnetmotor. Das 
Grundprinzip ist es erstmal überhaupt einen Magneten zu überwinden und 
auch Überschuss übrig zu haben.
Alles weitere macht dann erst Sinn.

Erläuterung:

Bisher waren die Magnete im Rotor und im Stator statisch. Diese habe ich 
nun dynamisch gemacht.
Das heißt sie drehen sich und passen sich immer gegenseitig mit dem 
Magentfeld an, da sie sich perfekt zueinander ausrichten. Das wiederum 
bedeutet, es wirkt immer die stärkste Kraft, aber das nur auf der 
Beschleunigungsseite. Hierbei gilt es zu klären, ob es einen 
grundlegenenden Unterschied der Kraft im Rotor macht, ob der Magnet 
statisch oder dynamisch ist. Bei einem statischen Magneten im Rotor 
wirken die Magnetkräfte in unterschiedlichen Richtungen, bei einem 
dynamischen Magneten immer nur eine Richtung, unzwar da, wo es gerade am 
stärksten ist, was wiederum auf die Achse wirkt. Hierbei bitte ich um 
Rat, wie hier die Unterschiede sind und ob ich bei einem dynamischen 
Magneten im Rotor eher Verlust mache oder ob das egal ist.

Wie erwähnt gilt die Ausrichtung der Magnete nur auf der 
Beschleunigungsseite. Ich habe entsprechende Wände eingebaut, sowohl im 
Rotor als auch Stator, um die Ausrichtung der Magnete für die Bremsseite 
zu verhindern. Hier sollte also nun weniger Magnetkraft wirken, als auf 
der Beschleunigungsseite. So jedenfalls die Idee.

Desweiteren habe ich den Statormagneten ein kleines bisschen nach rechts 
versetzt: siehe Bild "Mitte_verschoben"

Damit möchte ich erreichen, dass ich die volle Kraft der Magnete auf der 
Beschleunigungsseite erhalte, da die Entfernung des Rotors genau in der 
Mitte zum Stator am geringsten ist. Würden beide genau mittig zueinander 
liegen, wären 50% der geringsten Entfernung zueinander auf der 
Beschleunigungsseite und die anderen 50% auf der Bremsseite. Durch das 
Versetzen verändert sich der Prozentsatz zugunsten der 
Beschleunigungsseite.
Außerdem erhöht sich, durch das Versetzen, die Entfernung auf der 
Bremsseite zwischen Rotor und Stator schneller. Je weiter weg die 
Magnete sind, desto schwächer wird das Magnetfeld.
Wieviel Versatz hier richtig sind und ob es überhaupt Sinn macht, gilt 
es natürlich zu klären.

Euch sind vermutlich jegliche Projekte zum Magnetmotor bekannt und habt 
schon etliche Darstellungen gesehen. Kennt ihr Projekte die meiner Idee 
ähneln oder identisch sind?
Ich bin der festen Überzeugung, ein Ungleichgewicht geschaffen zu haben.
Ob das allerdings ausreicht, um daraus ein Perpetuum Mobile zu machen, 
sei dahin gestellt und müsste getestet werden.
Ich hoffe ihr habt soweit alles verstanden. Was haltet ihr davon? Ich 
freue mich über Meinungen, Fragen und Ratschläge.

: Verschoben durch Moderator
von H.Joachim S. (crazyhorse)


Lesenswert?

Steven schrieb:
> Ich bin der festen Überzeugung, ein Ungleichgewicht geschaffen zu haben.
Das gehört zu derartigen Versuchen
> Ob das allerdings ausreicht, um daraus ein Perpetuum Mobile zu machen,
> sei dahin gestellt und müsste getestet werden.
Muss man nicht

von Paule M. (martin_mu)


Lesenswert?

Steven, darf ich fragen, wie alt du bist?
Wie bist du auf das Thema gekommen?
Leider wird es nicht funktionieeren

von Axel S. (a-za-z0-9)


Lesenswert?

Verwendet dein Perpetuum Mobile auch "Analoge Elektronik und 
Schaltungstechnik"? Ich seh sie nicht. Falsches Forum erwischt?

@Moderation: bitte nach Offtopic oder gleich /dev/null verschieben!

von H.Joachim S. (crazyhorse)


Lesenswert?

youtube-Videos, da gehts bei perpetuum mobile immer um Magnete...
Gerne mit Heisskleber befestigt. Die schöpferische Leistung besteht 
immer nur darin den Energieeintrag mehr oder weniger gut zu 
verschleiern.

von Mani W. (e-doc)


Lesenswert?

Axel S. schrieb:
> Verwendet dein Perpetuum Mobile auch "Analoge Elektronik und
> Schaltungstechnik"? Ich seh sie nicht. Falsches Forum erwischt?

Eine superblöde Antwort!

von Matthias S. (Firma: matzetronics) (mschoeldgen)


Lesenswert?

Also, ich finde das toll. Da kommt einer, baut das PM und schon im 
nächsten Jahr ist er in Stockholm eingeladen. Und er kann sich ab da 
auch besseres Zeug hinter die Binde giessen als die Plörre, die in 
seinem Nick steht.

Muss er nur noch bauen...

: Bearbeitet durch User
von H. H. (hhinz)


Lesenswert?

Matthias S. schrieb:
> nächsten Jahr ist er in Stockholm eingeladen.

Beckomberga ist doch seit 30 Jahren geschlossen.

von Giovanni (sqrt_minus_eins)


Lesenswert?

Steven schrieb:
> Ich bin der festen Überzeugung,

Ich auch.

Lieber Steven,
Auch für Dich gilt:
* der Energieerhaltungssatz
* der 1. und 2. Hauptsatz der Wärmelehre
* die Maxwell-Gleichungen
* das Ohm'sche Gesetz
* und alle anderen Naturgesetze die sich seit >2000 Jahren gut bewährt 
haben.

Denk drüber nach, bevor du einen 1km langen Beitrag schreibst.

von Dieter D. (Firma: Hobbytheoretiker) (dieter_1234)


Lesenswert?

Nee, am Freitag mit dem Post schreiben nicht fertig geworden und nicht 
bis zum naechsten geschafft zu warten.

von Jan H. (j_hansen)


Lesenswert?

Steven schrieb:
> Ich hoffe ihr habt soweit alles verstanden. Was haltet ihr davon? Ich
> freue mich über Meinungen, Fragen und Ratschläge.

Warum schreibt du dazu einen Aufsatz und baust es nicht einfach und 
holst dir deinen Nobelpreis ab? Wird schon funktionieren, ist ja nicht 
so, dass das schon Millionen von Leuten versucht hätten und die 
Schöpfungshöhe deiner Idee ist ja auch enorm.

von Monk (roehrmond)


Lesenswert?

Steven schrieb:
> Was haltet ihr davon?

Magnetmotoren basieren auf der Idee, dass Magnete dauerhaft freie 
Energie liefern würden. Tun sie nicht, egal wie man sie dreht und 
wendet.

: Bearbeitet durch User
von Marcel V. (mavin)


Lesenswert?

Steven schrieb:
> Was haltet ihr davon? Ich freue mich über Meinungen, Fragen und
> Ratschläge.

Montiere nicht nur einen, sondern noch weitere Magnete am Umfang an, 
dann hast du mehr Power, damit sich das System selbst am Leben erhält 
und nicht nach einer halben Umdrehung gleich verhungert!

Mehr Magnete sorgen für ordentlich Drehzahl.

von Michael B. (laberkopp)


Lesenswert?

Steven schrieb:
> Was haltet ihr davon?

Die Finger.

Denn wir wissen, dass es nicht funktionieren kann.

Giovanni schreibt so schön

> Auch für Dich gilt:
> der Energieerhaltungssatz
> der 1. und 2. Hauptsatz der Wärmelehre
> die Maxwell-Gleichungen
> Denk drüber nach.

von Benjamin K. (bentschie)


Lesenswert?

Steven schrieb:

Ich versuche es mal inhaltlich,...

> Um einen Magnetmotor bauen zu können, ist die Hauptbedingung ein
> Ungleichgewicht im Magnetfeld zu finden bzw. zwischen der
> Beschleunigungsseite und der Bremsseite. Wenn ich es nun schaffe, die
> Beschleunigungsseite größer bzw. stärker als die Bremsseite zu bauen,
> habe ich ein Ungleichgewicht.

Und genau das wird und kann nicht funktionieren. Es ist kein 
Ungleichgewicht möglich. Ein Magnetfeld ist Quellenfrei, das bedeutet, 
alle Magnetfeldlinien sind immer geschlossene durchgehende Linien. Du 
kannst nichts auf einer Seite abschirmen. Die Magnetfeldlinien gehen 
dann eben woanders lang. Und wenn du das dann in die Summe deine Kräfte 
mit reinrechnest kommt eben die besagte Null bei raus.

von Volker B. (Firma: L-E-A) (vobs)


Lesenswert?

Steven schrieb:

> Ich hoffe ihr habt soweit alles verstanden.

Nein, für solche Romane hab' ich leider keine Zeit.

> Was haltet ihr davon? Ich

Es wird schon einen Grund geben warum bis heute kein Perpetuum Mobile
"erfunden" wurde. Im zarten Alter von ca. 15 Jahren wollte ich auch so 
etwas basteln: Ein Stabmagnet zieht ein Eisenstück an, das auf einer 
Wippe befestigt ist. Die Wippe kippt und dreht über eine Hebelmechanik 
dann den Magnet zur Seite, wobei der Mechanismus wie bei einem 
Schnappschalter arbeiten sollten, also eine Hysterese besitzt.

Der Aufbau hat mich Tage lang beschäftigt -- nur um festzustellen, dass 
das System stets in einer Position stehen bleibt, dem Energieminimum des 
Systems. Schade, schon wieder keine Chance auf den Nobelpreis :-(

> freue mich über Meinungen, Fragen und Ratschläge.

Lern Dich in ein FEM-Programm ein, z.B.: https://www.femm.info

Da gibt's eine lange Liste von Beispielen
https://www.femm.info/wiki/Examples
von denen sich vermutlich eines an Deine Idee anpassen lässt.

Du kannst das Drehmoment zwischen Rotor und Stator bei verschiedenen 
Rotorwinklen berechnen lassen. Dass lässt sich auch automatisieren über 
ein LUA-Skript.

Grüßle,
Volker

Nachtrag: Vor Jahren habe ich mit diesem Programm einen Schülerversuch 
entworfen und dafür ein auführliches, bebildertes Tutorial erstellt.
Die Schüler/innen aus der gymnasialen Mittelstufe konnten damit 
weitgehend selbständig arbeiten:
https://www.dr-bosch.com/downloads/FEMM_Magnetversuch_v4.pdf
Anschließend wurden die Berechnungen an einem einfachen Versuchsaufbau 
verifiziert.

: Bearbeitet durch User
von Wolfgang R. (Firma: www.wolfgangrobel.de) (mikemcbike)


Lesenswert?

Wetten, dass da nix mehr kommt?

von Florian (flori_n)


Lesenswert?

Mich würde noch interessieren, aus welchem konkreten Material diese 
Wände sein sollen. Damit meine ich nicht "Irgendein Material, das...", 
sondern eine konkrete Bezeichnung. Sollen die nur einen mechanischen 
oder einen magnetischen Effekt haben?

Monk schrieb:
> Steven schrieb:
>> Was haltet ihr davon?
>
> Magnetmotoren basieren auf der Idee, dass Magnete dauerhaft freie
> Energie liefern würden. Tun sie nicht, egal wie man sie dreht und
> wendet.
Ach, doch. Wenn man einen Magneten (schnell) dreht, kann man mit 
geschicktem Aufbau durchaus (elektrische) Energie zur freien Verfügung 
entnehmen. ;-)

von Giovanni (sqrt_minus_eins)


Lesenswert?

Volker B. schrieb:
> Lern Dich in ein FEM-Programm ein, z.B.: https://www.femm.info
>
> Da gibt's eine lange Liste von Beispielen
> https://www.femm.info/wiki/Examples
> von denen sich vermutlich eines an Deine Idee anpassen lässt.

Soweit ich gesehen habe gab es vom Autor schon einmal einen PM Beitrag 
zum Thema Kraft. Hat nichts geholfen.

Ich denke FEMM ist ein tolles Werkzeug. Man damit kann schnell schöne 
Bilder erzeugen. ABER Grundkennkenntnisse von FEM und Formulierungen 
sind schon notwendig um die Qualität der Ergebnisse deuten zu können (zB 
Maxwellscher Stress-Tensor, virtuelle Verschiebung, ...).

PS: https://www.dr-bosch.com/person.php  Gute Zusammenstellung der 
Bibliothek. Buch vom Andi Binder, alte Bücher vom Springer-Verlag, ...

von Volker B. (Firma: L-E-A) (vobs)


Lesenswert?

Giovanni schrieb:

> Ich denke FEMM ist ein tolles Werkzeug. Man damit kann schnell schöne
> Bilder erzeugen. ABER Grundkennkenntnisse von FEM und Formulierungen
> sind schon notwendig um die Qualität der Ergebnisse deuten zu können (zB
> Maxwellscher Stress-Tensor, virtuelle Verschiebung, ...).

Naja, um ein Beispiel zu modifizieren benötigt man eigentzlich kein 
tiefes Detailwissen, auch wenn's nicht schaden würde. Aber mit diesem 
Grundlagenwissen hat man dann  (leider) keine solchen "revolutionäre" 
Ideen mehr... :-)

> PS: https://www.dr-bosch.com/person.php  Gute Zusammenstellung der
> Bibliothek. Buch vom Andi Binder, alte Bücher vom Springer-Verlag, ...

Danke! :-)

Die Literaturliste gibt's im privaten Bereich meiner Homepage:
https://www.dr-bosch.com/volker/literatur/elmasch/node1.html

Auch für das Thema Magnetismus:
https://www.dr-bosch.com/volker/literatur/magnet/node1.html

Grüßle,
Volker

: Bearbeitet durch User
von Georg M. (g_m)


Angehängte Dateien:

Lesenswert?

Axel S. schrieb:
> Verwendet dein Perpetuum Mobile auch "Analoge Elektronik und
> Schaltungstechnik"?

von Matthias S. (Firma: matzetronics) (mschoeldgen)


Lesenswert?

Florian schrieb:
> Ach, doch. Wenn man einen Magneten (schnell) dreht, kann man mit
> geschicktem Aufbau durchaus (elektrische) Energie zur freien Verfügung
> entnehmen. ;-)

Jaja. Als ich noch klein war, wollte ich mir auch mal eine N-Maschine 
bauen. Aber da kommt nix...

von H. H. (hhinz)


Lesenswert?

Florian schrieb:
> Mich würde noch interessieren, aus welchem konkreten Material diese
> Wände sein sollen.

Trollium

von Steven (muellermilch)


Lesenswert?

Wolfgang R. schrieb:
> Wetten, dass da nix mehr kommt?

Wie hoch war dein Einsatz? ;)

Benjamin K. schrieb:
> Und genau das wird und kann nicht funktionieren. Es ist kein
> Ungleichgewicht möglich.

Es geht nicht darum das Magnetfeld abzuschirmen. Ich habe in meinem 
Motor nichts, was abschirmen soll. Es geht lediglich um die Verschiebung 
und Drehung der Position der Magnete. Magnete richten sich immer 
zueinander aus, wenn sie es können. Genau in dem Moment wirkt die 
stärkste Kraft. Diesen Effekt möchte mir auf der Beschleunigsseite zu 
nutzen machen und auf der Bremsseite blockieren.

Volker B. schrieb:
> Lern Dich in ein FEM-Programm ein

Ich habe mit FEMM schon ein bisschen rumgespielt und kenne mich ein 
bisschen aus. Wie soll ich aber einen Magneten messen, der sich um seine 
eigene Achse dreht und sich zum anderen Magneten ausrichtet? Reicht es, 
wenn ich da unterschiedliche Winkel messe? Schau dir doch gerne mal 
wenigstens das Gif oben an.

Volker B. schrieb:
>
Du kannst das Drehmoment zwischen Rotor und Stator bei verschiedenen
Rotorwinklen berechnen lassen. Dass lässt sich auch automatisieren über
ein LUA-Skript.

Wie das funktioniert, weiß ich leider nicht, würde ich aber gerne mal 
sehen.

Florian schrieb:
> Mich würde noch interessieren, aus welchem konkreten Material diese
> Wände sein sollen.

Die Wände sind aus Plastik oder anderem nicht magnetischem Material. Die 
Wände sollen lediglich die Ausrichtung der Magnete stoppen.

von Volker B. (Firma: L-E-A) (vobs)


Lesenswert?

Steven schrieb:

> Ich habe mit FEMM schon ein bisschen rumgespielt und kenne mich ein
> bisschen aus. Wie soll ich aber einen Magneten messen, der sich um seine
> eigene Achse dreht und sich zum anderen Magneten ausrichtet? Reicht es,
> wenn ich da unterschiedliche Winkel messe? Schau dir doch gerne mal
> wenigstens das Gif oben an.

Aus dem Bildchem werde ich nicht schlau. Die weißen Kringel sollen 
Drehachsen sein?

Dann dreh' jeden Magnet so lange um seine Achse, bis das Drehmoment an 
ihm 0 ist, ggf. mit einem kleinen LUA-Skript.

> Volker B. schrieb:
> Du kannst das Drehmoment zwischen Rotor und Stator bei verschiedenen
> Rotorwinklen berechnen lassen. Dass lässt sich auch automatisieren über
> ein LUA-Skript.

> Wie das funktioniert, weiß ich leider nicht, würde ich aber gerne mal
> sehen.

Ich guck' mir heute Abend meine alten Rechnungen an. Auswendig weiß ich 
das nicht mehr, da die letzte FEMM-Anwedung bereits mehrere Jahre 
zurückliegt.

Grüße,
Volker

von Steven (muellermilch)


Lesenswert?

Volker B. schrieb:
> Aus dem Bildchem werde ich nicht schlau. Die weißen Kringel sollen
> Drehachsen sein?

Hallo Volker, ja genau. Wenn du nicht auf das Bild klickst, sondern 
direkt auf den Namen "mm.gif" dann sollte das Gif funktionieren und du 
siehst eine Bewegung.

Volker B. schrieb:
> Dann dreh' jeden Magnet so lange um seine Achse, bis das Drehmoment an
> ihm 0 ist, ggf. mit einem kleinen LUA-Skript.

Oh, das klingt ja interessant. Wie genau soll das aber mit 2 Magneten 
funktionieren, die sich immer zueinander perfekt ausrichten? Dieses 
zueinander ausrichten stelle ich mir sehr schwierig in FEMM vor, 
vorallem wenn sich dann noch der Rotor dreht und die Entfernung zwischen 
den Magneten sich verändert.

Volker B. schrieb:
> Ich guck' mir heute Abend meine alten Rechnungen an. Auswendig weiß ich
> das nicht mehr, da die letzte FEMM-Anwedung bereits mehrere Jahre
> zurückliegt.

Perfekt, danke dir.

von Rolf (rolf22)


Lesenswert?

Matthias S. schrieb:
> Als ich noch klein war, wollte ich mir auch mal eine N-Maschine
> bauen.

Als ich noch klein war, kostete ein NF-Transistor im Bastlerladen zehn 
D-Mark. Für mich waren das 5 elterliche Monats-Taschengelder – praktisch 
unbezahlbar. Zum Glück konnte ich mit Kohlenschleppen in den vierten 
Stock genug hinzuverdienen, und ein selbst gebautes Radio in der 
Seifendose musste her. Es war ja die Röhren-Ära, da war ich mit so was 
King in der Schulklasse.

Die "Lösung":
Eine Germanium-Diode kostete nur einen Bruchteil davon, und ich hatte 
alte Bücher mit Bildern des Aufbaus von Spitzentransistoren.

Pfiffig, wie ich war, nahm ich zwei Dioden und lötete die 
Substrat-Anschlüsse zusammen. Zwei Metallspitzen auf einem gemeinsamen 
Kristall, das müsste doch einen Spitzentransistor ergeben! Tja, 
Halbleiterphysik war mir so unbekannt wie Energie, Kräfte und 
Magnetfelder so manchem heutigen Perpetuum-mobile-Erfinder.

von Christian B. (luckyfu)


Lesenswert?

Steven schrieb:
> Genau in dem Moment wirkt die
> stärkste Kraft. Diesen Effekt möchte mir auf der Beschleunigsseite zu
> nutzen machen und auf der Bremsseite blockieren.

Was genau hast du an dieser Aussage:

Benjamin K. schrieb:
> Ein Magnetfeld ist Quellenfrei, das bedeutet,
> alle Magnetfeldlinien sind immer geschlossene durchgehende Linien. Du
> kannst nichts auf einer Seite abschirmen. Die Magnetfeldlinien gehen
> dann eben woanders lang. Und wenn du das dann in die Summe deine Kräfte
> mit reinrechnest kommt eben die besagte Null bei raus.

nicht verstanden?

BTW: Man kann Magnete nicht in Nord- und Südpol aufteilen und getrennt 
voneinander einbauen. Sobald du einen Magnet teilst, hast du wieder 
kleine Magnete mit der selben Ausrichtung wie vorher der Große. Es gibt 
keinen N und keinen S Magneten die irgendwie zusammengeklebt wurden.


Steven schrieb:
> Genau in dem Moment wirkt die
> stärkste Kraft. Diesen Effekt möchte mir auf der Beschleunigsseite zu
> nutzen machen und auf der Bremsseite blockieren.
BTW: Was du natürlich machen kannst ist: das gewünschte Verhalten mit 
einem steuerbaren Magneten zu erreichen. Dieser wird üblicherweise 
gebaut, indem man Draht um ein Ferromagnetisches Material wickelt und 
Strom hindurchschickt.

Das Ergebnis ist dann ein schnöder Elektromotor. Hat halt den Nachteil, 
dass der nicht besonders Innovativ erscheint, aber den großen Vorteil, 
dass er Funktioniert.

: Bearbeitet durch User
von Marcel V. (mavin)


Angehängte Dateien:

Lesenswert?

Steven schrieb:
> Wenn ich es nun schaffe, die Beschleunigungsseite größer bzw. stärker
> als die Bremsseite zu bauen

Leider stehen sich die Pole kurz vor der Abrisskante, also bei maximaler 
Bremsung am dichtesten gegenüber (blauer Abstand "A"), so dass die 
gesamte vorherige Beschleunigung komplett wieder abgebremst wird.

Man müsste direkt an der Abrisskante ein Eisen-, Nickel-, oder 
Kobaltblech genau zwischen den Abstand "A" anbringen, so dass das 
Magnetfeld noch abrupter abgerissen wird, damit erst gar keine 
Bremsverzögerung eintreten kann!

von Volker B. (Firma: L-E-A) (vobs)


Lesenswert?

Steven schrieb:

> Hallo Volker, ja genau. Wenn du nicht auf das Bild klickst, sondern
> direkt auf den Namen "mm.gif" dann sollte das Gif funktionieren und du
> siehst eine Bewegung.

OK. Das wird so nicht funktionieren, denn das Trennen der Magnete kostet 
Energie, auch wenn diese nicht (exakt) miteinander fluchten.

> Volker B. schrieb:
>> Dann dreh' jeden Magnet so lange um seine Achse, bis das Drehmoment an
>> ihm 0 ist, ggf. mit einem kleinen LUA-Skript.
>
> Oh, das klingt ja interessant. Wie genau soll das aber mit 2 Magneten
> funktionieren, die sich immer zueinander perfekt ausrichten? Dieses
> zueinander ausrichten stelle ich mir sehr schwierig in FEMM vor,
> vorallem wenn sich dann noch der Rotor dreht und die Entfernung zwischen
> den Magneten sich verändert.

1. Startposition annehmen
2. Drehmomente auf die drehbaren Körper berechnen
3. Alle Körper um einen kleinen Winkel in Richtung der Drehmomente 
bewegen, sofern dieses nicht durch einen Anschlag verhindert wird.
4. Weiter mit 2.

Aber, wie oben beschrieben: Das System wird in der Postion verharren, wo
die Kraft ziwschen den Magneten am größten ist, d.h. der Abstand der 
Polenden am geringsten ist bzw. die "magnetische Flussröhre", die die 
beiden Poloberflächen verbindet, den höchsten magnetischen Leitwert 
besitzt.

Es gibt bei dieser Anordnung keine Abstoßung zwischen den Magneten, 
außer Du drehst sie so, dass sich zwei gleiche Pole gegenüber stehen -- 
aber das kostet Energie!

Stell Dir die magnetischen Feldlinien wie gespannte Gummibänder vor.

Grüßle,
Volker

: Bearbeitet durch User
von Steven (muellermilch)


Lesenswert?

Christian B. schrieb:
> Man kann Magnete nicht in Nord- und Südpol aufteilen und getrennt
> voneinander einbauen.

Du missverstehst etwas. Ich teile den Magneten nicht in Nord- und 
Südpol, sondern nur einen Pol in 2 Hälften auf, aber auch nur sinngemäß.
Schau dir doch dazu bitte das Bild "TeilungMagnetfeld" nochmal genau an.

von Marcel V. (mavin)


Lesenswert?

Volker B. schrieb:
> Das wird so nicht funktionieren, denn das Trennen der Magnete kostet
> Energie, auch wenn diese nicht (exakt) miteinander fluchten.

Schade dass du schon was geschrieben hast, ich wollte gerade meinen 
Beitrag wieder löschen, um mir meine Idee mit dem 
Absorptionszwischenblech patentieren zu lassen.

Ich werde jetzt aber trotzdem noch versuchen, dass mir mein Arbeitgeber 
heute Nachmittag frei gibt, damit ich jetzt noch eben schnell zum 
Patentamt fahren kann, sonst tut es womöglich hier noch ein anderer 
Mitleser!

von Volker B. (Firma: L-E-A) (vobs)


Lesenswert?

Marcel V. schrieb:

> Schade dass du schon was geschrieben hast, ich wollte gerade meinen
> Beitrag wieder löschen, um mir meine Idee mit dem
> Absorptionszwischenblech patentieren zu lassen.
>
> Ich werde jetzt aber trotzdem noch versuchen, dass mir mein Arbeitgeber
> heute Nachmittag frei gibt, damit ich jetzt noch eben schnell zum
> Patentamt fahren kann, sonst tut es womöglich hier noch ein anderer
> Mitleser!

Das ist jetzt nicht Dein Ernst oder?
Spar' Dir den Aufwand. Das funktioniert nicht, denn Blech und Magnet 
ziehen sich ebenfalls an.

Außerdem ist eine Vorabveröffentlichung "schädlich" für eine 
Patentanmeldung. Dein "Wissen" wäre damit bereits zum Stand der 
"Technik" geworden, wenn's denn funktionieren würde...

Grüßle,
Volker

von Steven (muellermilch)


Lesenswert?

Marcel V. schrieb:
> Leider stehen sich die Pole kurz vor der Abrisskante, also bei maximaler
> Bremsung am dichtesten gegenüber (blauer Abstand "A"), so dass die
> gesamte vorherige Beschleunigung komplett wieder abgebremst wird.

Volker B. schrieb:
> Das System wird in der Postion verharren, wo
> die Kraft ziwschen den Magneten am größten ist, d.h. der Abstand der
> Polenden am geringsten ist bzw. die "magnetische Flussröhre", die die
> beiden Poloberflächen verbindet, den höchsten magnetischen Leitwert
> besitzt.

Ich antworte euch beiden auf eure Beiträge, da die sich ähneln.

Kennt ihr die Abschussrampe, wo eine Kugel auf einer Magnetbahn rollt?
(Quelle: https://www.supermagnete.de/Magnetanwendungen/Abschussrampe)
Das Prinzip ist ähnlich. Dabei spielt es natürlich keine Rolle, wie 
wieviele Magnete im Stator sind.
Betrachten wir folgende Situation. Der Startpunkt der Kugel ist 7cm vom 
ersten Magneten entfernt. Die Kugel rollt los und würde ohne 
Reibungsverluste exakt 7cm hinter dem letzten Magneten landen und wieder 
zurück rollen. Korrekt?
So stelle ich es mir mit meinem Motor vor. Ich habe einen Startpunkt des 
Rotors, wo die Magnete sich gegenseitig anziehen. Dieser Startpunkt 
liegt als Beispiel 7cm vom Mittelpunkt des Statormagneten entfernt. Das 
ist meine Beschleunigungsstrecke. Auf dieser Strecke werden innerhalb 
dieser 7cm Energie erzeugt. Nun ist mein Rotor genau am Mittelpunkt, 
hier beginnt der Bremsweg. Ohne Reibungsverluste wird mein Rotor genau 
diese 7cm auch als Endpunkt erreichen.

Warum genau, sollte mein Rotor genau in der Mitte stehen bleiben, wo die 
stärkste Kraft wirkt? Das passiert ja bei der Kugel auch nicht. Die 
Kugel rollt solange hin und her, bis die Energie durch die 
Reibungsverluste umgewandelt ist. Das gleiche passiert mit meinem Rotor. 
Der Unterschied ist, dass ich behaupte, meine Beschleunigsseite hat mehr 
Energie, als meine Bremsseite, da andere Magnetkräfte innerhalb der 
Strecke wirken.
Ob allerdings diese ausreichen, um den Reibungsverlust auszugleichen, 
weiß ich nicht.

: Bearbeitet durch User
von Wolfgang R. (Firma: www.wolfgangrobel.de) (mikemcbike)


Lesenswert?

Komisch, dass trotz tausender sinnloser YouTube Videos über nicht 
funktionierende Magnetmotoren immer noch jemand um die Ecke kommt, der 
jatzt aber wirklich die Lösung für ein Perpetuum Mobile gefunden haben 
will...

Ein Magnet ist keine Energiequelle!
Eine Feder ist auch keine Energiequelle!

Ich kann da nicht mehr rausholen, als ich vorher mechanisch reingesteckt 
habe.

von Michael B. (laberkopp)


Lesenswert?

Marcel V. schrieb:
> Man müsste

Man müsste bei den Grundlagen der Physik in der Schule aufgepasst haben.

Das scheint aus der Mode zu sein.

Dummheit rulez ?

von ●DesIntegrator ●. (Firma: FULL PALATINSK) (desinfector) Benutzerseite


Lesenswert?

wieviel Effizienz gibt es bei Elektromotoren
eigentlich noch hinzu zu gewinnen?

Man ist doch jetzt so bei 97%

von Wolfgang R. (Firma: www.wolfgangrobel.de) (mikemcbike)


Lesenswert?

Steven schrieb:
> Der Unterschied ist, dass ich behaupte, meine Beschleunigsseite hat mehr
> Energie, als meine Bremsseite, da andere Magnetkräfte innerhalb der
> Strecke wirken.

Genau DAS ist deine Fehlannahme.

von Steven (muellermilch)


Lesenswert?

Wolfgang R. schrieb:
> Genau DAS ist deine Fehlannahme.

Bitte um Erläuterung, warum das so ist.
Wirken keine anderen Kräfte, wenn die Magnete unterschiedlich zueinander 
ausgerichtet sind?

von Wolfgang R. (Firma: www.wolfgangrobel.de) (mikemcbike)


Lesenswert?

Steven schrieb:
> Wirken keine anderen Kräfte, wenn die Magnete unterschiedlich zueinander
> ausgerichtet sind?

Die Kräfte mögen unterschiedlich sein, dann sind es aber die Wege auch. 
Letzendlich wirst du für das Eintauchen in das Magnetfeld (bei 
Anziehung) genau jene Energie erhalten, die du zum Verlassen desselben 
wieder benötigst. Abzüglich der Reibungsverluste.

von Wolfgang R. (Firma: www.wolfgangrobel.de) (mikemcbike)


Lesenswert?

Du kannst das jetzt einfach glauben, oder du baust ein Muster auf und 
überzeugst dich selbst. Die Wahrscheinlichkeit, die Physik des 
Universums zu widerlegen ist sehr gering, quasi Null.

von Giovanni (sqrt_minus_eins)


Lesenswert?

Steven (der Verfasser) hat sich mit dem Thema schon früher im Detail 
beschäftigt, und hat uns jetzt seine Erkenntnisse in diesem Beitrag 
präsentiert. Das ist soweit OK.

Beitrag "Elektromagnet = Neodym-Magnet"

Vielleicht sollte man eine KI befragen. Als unabhängigen Experten. Durch 
meine Vorkenntnisse erkläre ich mich hiermit als befangen.

: Bearbeitet durch User
von Steven (muellermilch)


Lesenswert?

Wolfgang R. schrieb:
> Letzendlich wirst du für das Eintauchen in das Magnetfeld (bei
> Anziehung) genau jene Energie erhalten, die du zum Verlassen desselben
> wieder benötigst.

Das ist doch aber physikalisch unlogisch, wenn auf beiden Seiten 
unterschiedliche Kräfte wirken. Die Energie kann nicht gleich sein.
Wenn eine Seite 10kg hat und die andere Seite 9kg, dann ergibt das nicht 
0.

Wolfgang R. schrieb:
> Die Wahrscheinlichkeit, die Physik des
> Universums zu widerlegen ist sehr gering, quasi Null.

Ich habe nie behauptet die Physik zu widerlegen oder die Lösung für ein 
Perpetuum Mobile gefunden zu haben. Ich behaupte lediglich, ein 
Ungleichgewicht gefunden zu haben, dass ist aber noch lange kein Grund 
für ein Perpetuum Mobile.

von Wolfgang R. (Firma: www.wolfgangrobel.de) (mikemcbike)


Lesenswert?

Steven schrieb:
> Ich behaupte lediglich, ein
> Ungleichgewicht gefunden zu haben, dass ist aber noch lange kein Grund
> für ein Perpetuum Mobile.

Ich hatte deinen Eingangsthread so verstanden, dass du dieses 
Ungleichgewicht zum Antrieb eines Motors verwenden möchtest. Wo ist da 
der Unterschied zum PM?

von 🍅🍅 🍅. (tomate)


Lesenswert?

Sehr Interessant, wieviel freie Energie generiert denn der Gerät?

Magie ist Physik/Wollen, muss man wissen!

von Steven (muellermilch)


Lesenswert?

Wolfgang R. schrieb:
> Ich hatte deinen Eingangsthread so verstanden, dass du dieses
> Ungleichgewicht zum Antrieb eines Motors verwenden möchtest. Wo ist da
> der Unterschied zum PM?

Die Faktoren für ein PM sind viel zu komplex. Letzendlich kommt es auf 
ein PM hinaus, da kann ich mich drehen und wenden wie ich will.
Solange ich aber nicht weiß, ob es ein Ungleichgewicht gibt und wieviel 
es überhaupt bringt, macht es keinen Sinn annähernd über ein PM zu 
sprechen.

von Robert M. (r_mu)


Lesenswert?

Konzeptionell kann man die Magnete mit Federn ersetzen, da wirds 
vielleicht dann klarer dass eine gespannte Feder immer mit der selben 
Kraft in beide Seiten drückt. Mit den Magneten verhält es sich genauso, 
ganz egal wie man die anordnet, oder vorspannt.

von Oliver S. (oliverso)


Lesenswert?

Steven schrieb:
> Solange ich aber nicht weiß, ob es ein Ungleichgewicht gibt und wieviel
> es überhaupt bringt, macht es keinen Sinn annähernd über ein PM zu
> sprechen.

Es macht grundsätzlich überhaupt keinen Sinn, auch nur annähernd über 
ein PM zu sprechen, egal, ob du etwas weißt oder nicht.

Geht nicht, gibts nur in der virtuellen Realität auf youtube.

Oliver

von Wolfgang R. (Firma: www.wolfgangrobel.de) (mikemcbike)


Lesenswert?

Steven schrieb:
> Letzendlich kommt es auf
> ein PM hinaus

Steven schrieb:
> macht es keinen Sinn annähernd über ein PM zu
> sprechen

Es gibt kein PM.
Es gibt keinen Motor, der sich ohne externe Energiezufuhr selbst am 
laufen hält, geschweige denn, dem man noch Energie entnehmen könnte.
Dein postuliertes Ungleichgewicht gibt es nicht. Energetisch ist da 
immer ein Punkt, in dem sich die wirkenden Kräfte aufheben - auch wenn 
der halt nicht in der Mitte liegt.
Die Diskussion ist sinnlos. Bau ein Modell auf und prüfe selbst.

Und wie bei fast allen Freie-Energie-Hoffenden wird dann am Ende die nie 
versiegende Hoffnung stehen, dass es mit ein paar kleinen Optimierungen 
dann doch noch läuft. Wird es aber nicht. Nie. Da steht schon der 
Energieerhaltungssatz dagegen.

von Robert M. (r_mu)


Lesenswert?

Abgesehen von der Absurdität der Vorstellung "etwas" (nützliche Energie) 
aus "nichts" zu erschaffen würde die Existenz eines perpetuum mobile im 
besten Fall die globale Erwärmung und im Endeffekt den Hitzetod des 
Universums (massiv) beschleunigen -- die Idee dass man mit Energie 
ungestraft allzu verschwenderisch umgehen kann ist falsch.

von Wolfgang R. (Firma: www.wolfgangrobel.de) (mikemcbike)


Lesenswert?

Robert M. schrieb:
> würde die Existenz eines perpetuum mobile im
> besten Fall die globale Erwärmung und im Endeffekt den Hitzetod des
> Universums (massiv) beschleunigen

Gottseidank geht das ja eben nicht... ;-)

von Robert M. (r_mu)


Lesenswert?

Bei der Gelegenheit will ich auch noch eine Warnung vor Experimenten mit 
großen Supermagneten anbringen, die Dinger sind ab einer gewissen Größe 
wirklich gefährlich, "sehen" sich auch in 1-2 Meter Entfernung und 
fliegen gerne aufeinander zu. Ist dann ein Finger dazwischen kanns je 
nach Größe der Magnete auch auf eine Amputation rauslaufen wenn die 
Keramiken beim Zerquetschen des Fingers zerbröseln.

von Wolfgang R. (Firma: www.wolfgangrobel.de) (mikemcbike)


Lesenswert?

Was mit kleinen Magneten nicht fuktioniert, funktioniert mit großen auch 
nicht... Overunity ist nicht skalierbar.

von Florian (flori_n)


Lesenswert?

Steven schrieb:
> Das ist doch aber physikalisch unlogisch, wenn auf beiden Seiten
> unterschiedliche Kräfte wirken. Die Energie kann nicht gleich sein.
Doch, solange der Weg bzw. Zeit unterschiedliche ist.

Rein logisch gesehen ist es aber auch unlogisch, dem 
Energieerhaltungssatz zu widersprechen und dann aber mit physikalisch 
unlogisch zu argumentieren.

von Steven (muellermilch)


Lesenswert?

Florian schrieb:
> Doch, solange der Weg bzw. Zeit unterschiedliche ist.

Richtig, deswegen nehme ich ja als Referenz für die Berechnung den 
gleichen Weg. Ist der Weg auf der Bremsstrecke länger als auf der 
Beschleunigungsstrecke, habe ich ein Ungleichgewicht.

von Ralf X. (ralf0815)


Lesenswert?

Steven schrieb:
> Florian schrieb:
>> Doch, solange der Weg bzw. Zeit unterschiedliche ist.
>
> Richtig, deswegen nehme ich ja als Referenz für die Berechnung den
> gleichen Weg. Ist der Weg auf der Bremsstrecke länger als auf der
> Beschleunigungsstrecke, habe ich ein Ungleichgewicht.

Du vergisst die jeweiligen Zeiten/Winkelgeschwindigkeiten einzubeziehen.

von Robert M. (r_mu)


Lesenswert?

Das mit FEMM zu simulieren ist ja auch ziemlich sinnbefreit. Aus den 
selben Grundlagen auf denen FEMM basiert kann man auch herleiten, dass 
der Magnetmotor nicht funktionieren kann. (das Integral der Kräfte auf 
den Magneten entlang jeder geschlossenen Bahn, die also zurück zum 
Ausgangspunkt kommt, was für eine "ewige" Rotation ja nötig wöre, immer 
0 sein wird)

Wenn man also schon der Meinung ist, dass sowas wie der Magnetmotor 
funktionieren kann, brauch ich dazu ein bisher unbekanntes Phänomen, 
welches natürlich auch in existierender FEM-Software nicht abgebildet 
sein kann.

von Wolfgang R. (Firma: www.wolfgangrobel.de) (mikemcbike)


Lesenswert?

Robert M. schrieb:
> welches natürlich auch in existierender FEM-Software nicht abgebildet
> sein kann.

Genau - deswegen bleibt nur der Aufbau eines Funktionsmusters.

von Steven (muellermilch)


Lesenswert?

Ralf X. schrieb:
> Du vergisst die jeweiligen Zeiten/Winkelgeschwindigkeiten einzubeziehen.

Was genau meinst du damit? Kannst diese Aussage bitte mal konkret auf 
mein Motor anwenden, damit ich es besser verstehe.

von Robert M. (r_mu)


Lesenswert?

Steven schrieb:
> Was genau meinst du damit? Kannst diese Aussage bitte mal konkret auf
> mein Motor anwenden, damit ich es besser verstehe.

Es gibt kein Ungleichgewicht. Das Ding verhält sich wie ein Pendel. Ein 
Pendel kann ich anstossen, dann pendelts hin und her bis die Energie als 
Reibung an der Aufhängung und mit der Luft verbraucht ist.

Genauso hält es sich mit allen Magnetmotoren. Die kann ich "aufziehen" 
und dann loslassen, dann dreht sich das Ding bis die Energie vom 
"Aufziehen" über Reibung in Wärme umgewandelt ist und dann steht das 
Ding.

von Uwe S. (bullshit-bingo)


Lesenswert?

Steven, wie du siehst, können sie dir auch nicht erklären, warum es 
nicht geht. Sie können dich nur als dumm hinstellen, irgendwelche 
unpassenden phys. Gesetze bemühen, usw.. So weit, so übel, aber leider 
inzwischen normal. Sei dir gewiss, daß sie keinen Deut mehr Wissen 
haben! Sie sind nur  bessere Schauspieler und Selbstdarsteller, die auf 
dahergelaufene TOs warten, an denen sie ihre Daseinsberechtigung 
aufpolieren können.

Nehmen wir mal die Animation als Vorlage. Dann beginnt es damit, daß 
sich die Magnete aus der Entfernung anziehen. Hier ist die Kraft noch 
gering, aber der Winkel der Krafteinwirkung auf den Rotor recht gut. 
Später steigert sich die Anziehungskraft deutlich, aber der Winkel wird 
immer schlechter, was trotz hoher Kraft wieder zu kaum Drehmoment führt. 
Unterm Strich kommt also während der Anziehungsphase kaum was bei rum. 
Müssen die Magnete hingegen getrennt werden, so ist dazu eine hohe Kraft 
nötig, und da sie sich nun nicht mehr drehen können, ist auch der Winkel 
nahezu ideal zur Verzögerung. Die Summe ist dabei null. Das ist die 
Erklärung hier.

Du kannst ganz unterschiedliche "Magnetmotoren" erdenken, es läuft immer 
auf dasselbe hinaus. Nur manchmal ist offensichtlicher, manchmal 
weniger, warum es nicht klappt. Vergeude daher bitte nicht weiter deine 
Zeit mit diesem Unfug. Die Welt hat wahrlich genug irdische Probleme, es 
ist nicht sinnvoll, auf Zauberei zu setzen.

von John B. (craftsman)


Lesenswert?

Steven schrieb:
> Der Unterschied ist, dass ich behaupte, meine Beschleunigsseite hat mehr
> Energie, als meine Bremsseite, da andere Magnetkräfte innerhalb der
> Strecke wirken.

Das kann schon sein, ABER damit das der Fall ist, musst du die Anordnung 
ändern. UND das Verändern der Anordnung kostet genau diese Menge an 
Energie, die du meinst gewonnen zu haben. Da bleibt nichts über.

von Ralf X. (ralf0815)


Lesenswert?

Steven schrieb:
> Ralf X. schrieb:
>> Du vergisst die jeweiligen Zeiten/Winkelgeschwindigkeiten einzubeziehen.
>
> Was genau meinst du damit? Kannst diese Aussage bitte mal konkret auf
> mein Motor anwenden, damit ich es besser verstehe.

Du hast da ein "Konstrukt" und keinen Motor!
Nachdem Du das Rad einmal in Rotation gesetzt hast, wird sich die 
Winkelgeschwindigkeit auch bei Vernachlässigung der Reibung in jedem 
Punkt einer Umdrehung ändern, also entweder eine Beschleunigung oder 
Verzögerung aufweisen.
Werden konstruktionsbedingt unterschiedliche Wegstrecken für 
Beschleunigung und Verzögerung erzwungen, ergeben sich auch 
unterschiedliche Zeiten und Temporärleistungen.
Das Integral der Leistung über eine Umdrehung ergibt dann wieder null.

von Dieter D. (Firma: Hobbytheoretiker) (dieter_1234)


Lesenswert?

John B. schrieb:
> musst du die Anordnung ändern. UND das Verändern der Anordnung kostet
> genau diese Menge an Energie,

Die Energie kommt sicherlich aus dem Bauch heraus. ;o)

Genauer gesagt, Bierwampe.

von Mark S. (voltwide)


Lesenswert?

Dieter D. schrieb:
> Die Energie kommt sicherlich aus dem Bauch heraus. ;o)
Witzig bist Du also auch nicht.

von Steven (muellermilch)


Lesenswert?

Uwe S. schrieb:
> aber der Winkel wird
> immer schlechter

Warum wird bei der Winkel schlechter? Genau das ist ja das was auf der 
Beschleunigungsseite wichtig ist, der Winkel passt sich immer perfekt 
zueinander an, bis der Bremsweg beginnt. Erst dann verändert sich dieser 
Winkel negativ. Die Magnete sind innerhalb der Beschleunigungsstrecke 
immer ideal zueinander ausgerichtet.

Ralf X. schrieb:
> Du hast da ein "Konstrukt" und keinen Motor!

Da hast du natürlich recht!

Ralf X. schrieb:
> Nachdem Du das Rad einmal in Rotation gesetzt hast, wird sich die
> Winkelgeschwindigkeit auch bei Vernachlässigung der Reibung in jedem
> Punkt einer Umdrehung ändern, also entweder eine Beschleunigung oder
> Verzögerung aufweisen.
> Werden konstruktionsbedingt unterschiedliche Wegstrecken für
> Beschleunigung und Verzögerung erzwungen, ergeben sich auch
> unterschiedliche Zeiten und Temporärleistungen.
> Das Integral der Leistung über eine Umdrehung ergibt dann wieder null.

Eine Umdrehung macht in meiner Konstruktion erstmal noch keinen Sinn. Im 
Prinzip geht es nur um eine definierte Wegstrecke. Einmal um die 
Beschleunigungsstrecke und einmal um die Bremsstrecke.
Das es unterschiedliche Geschwindigkeiten innerhalb dieser Strecken 
gibt, ist soweit klar. Was das nun genau mit der Winkelgeschwindigkeit 
auf sich hat, verstehe ich leider nicht.
Am Ende muss ich doch nur wissen, ob die Bremsstrecke länger war als die 
Beschleunigungsstrecke.

von Ralf X. (ralf0815)


Lesenswert?

Steven schrieb:
> Eine Umdrehung macht in meiner Konstruktion erstmal noch keinen Sinn. Im
> Prinzip geht es nur um eine definierte Wegstrecke. Einmal um die
> Beschleunigungsstrecke und einmal um die Bremsstrecke.
> Das es unterschiedliche Geschwindigkeiten innerhalb dieser Strecken
> gibt, ist soweit klar. Was das nun genau mit der Winkelgeschwindigkeit
> auf sich hat, verstehe ich leider nicht.
> Am Ende muss ich doch nur wissen, ob die Bremsstrecke länger war als die
> Beschleunigungsstrecke.

Stell Dir einfach mal vor, Du lässt ein Rad oder Kugel ohne Luft- und 
Rollreibungsverlusten eine schräge Ebene von 45° 
(Beschleunigungsstrecke) runterrollen  und anschliessend eine 10° 
Steigung (Bremsstrecke) wieder nach oben bis auf die gleiche Höhe der 
Ausgangssituation.
Deine Bremsstrecke ist also bedeutend länger, genauso die Zeit, die dort 
verbracht wird, aber das energetische Endergebnis ist null.
Ebenso, wenn Du den Vorgang umdrehst.

von Uwe S. (bullshit-bingo)


Lesenswert?

Steven schrieb:
> Die Magnete sind innerhalb der Beschleunigungsstrecke
> immer ideal zueinander ausgerichtet.

Ich meine die Richtung der Kraft, die auf den Rotor wirkt, aber auch 
tatsächlich in Drehmoment verwandelt werden kann. Dieses Produkt ist 
maximal nur an einem Punkt, und zwar genau dann, wenn die Magnete 
gestoppt sind, der Rotor aber weiterdrehen will. Leider wirkt es dann in 
die falsche Richtung.

von H.Joachim S. (crazyhorse)


Lesenswert?

Es lohnt sich einfach nicht über irgendwelche theoretischen Behauptungen 
dieser Art nachzudenken.
Nachdenken kann man dann, wenn ein Scheinprototyp dasteht. Das zu 
entlarven kann sogar Spass machen wenn es gut gemacht ist. Alles andere 
ist verlorene Zeit, weil die "Erfinder" eh keine Argumente zur Kenntnis 
nehmen. Eigentlich kommt dann auch immer irgendwas von Bedrohungen durch 
die kommerzielle Energiewirtschaft oder aufgekauften Patenten, die in 
der Schublade verschimmeln.

von Monk (roehrmond)


Lesenswert?

H.Joachim S. schrieb:
> Alles andere ist verlorene Zeit, weil die "Erfinder"
> eh keine Argumente zur Kenntnis nehmen.

Darauf wollte ich auch gerade hinweisen. Niemand konnte bisher ein 
Perpetuum mobile bauen, und das ist auch wissenschaftlich begründet. Wer 
mit dem Gedanken "ich kann es aber doch" kommt, der ist für 
wissenschaftlich basierte Erklärungen wahrscheinlich nicht empfänglich, 
denn er hat sich bereits dagegen entschieden.

> Eigentlich kommt dann auch immer irgendwas von Bedrohungen durch
> die kommerzielle Energiewirtschaft oder aufgekauften Patenten, die
> in der Schublade verschimmeln.

Ich kenne jemanden, der tausende Euro in die Idee versenkte, 
Benzin-Motoren mit Wasser effizienter zu machen. Angeblich wollte er das 
einige male vor Publikum vorführen, wurde aber immer kurz vorher von 
höheren Mächten daran gehindert. Er gab die Idee letztendlich auf, als 
deutliche Anzeichen Verfolgungswahn aufkamen. Traurige Sache, aber 
leider ein typischer Fall.

: Bearbeitet durch User
von Ralf X. (ralf0815)


Lesenswert?

H.Joachim S. schrieb:
> Es lohnt sich einfach nicht über irgendwelche theoretischen Behauptungen
> dieser Art nachzudenken.
> Nachdenken kann man dann, wenn ein Scheinprototyp dasteht. Das zu
> entlarven kann sogar Spass machen wenn es gut gemacht ist. Alles andere
> ist verlorene Zeit, weil die "Erfinder" eh keine Argumente zur Kenntnis
> nehmen. Eigentlich kommt dann auch immer irgendwas von Bedrohungen durch
> die kommerzielle Energiewirtschaft oder aufgekauften Patenten, die in
> der Schublade verschimmeln.

Aktuell dröhnt die DZ-Bank die Werbefenster aller Sender mit ihrer 
Perpetuum Mobile Werbung zu.
https://www.youtube.com/watch?v=XTRH_iZ0ap8
Natürlich ruft so ein Humbug wieder mal einen Run der kleinen Erfinder 
ohne physikalischen Grundlagen auf den Plan.
Viele Finanzinstitute haben es halt auf strohdumme Kunden abgesehen..

von H.Joachim S. (crazyhorse)


Lesenswert?

Eieiei,das ist ja übel...
Sieht auf den den ersten Blick so logisch aus :-)

von Günter L. (Firma: Privat) (guenter_l)


Lesenswert?

von Steven schrieb:
>Ralf X. schrieb:
>> Du vergisst die jeweiligen Zeiten/Winkelgeschwindigkeiten einzubeziehen.

>Was genau meinst du damit? Kannst diese Aussage bitte mal konkret auf
>mein Motor anwenden, damit ich es besser verstehe.

Energie = Arbeit = Leistung mal Zeit

von Florian (flori_n)


Lesenswert?

H.Joachim S. schrieb:
> Es lohnt sich einfach nicht über irgendwelche theoretischen Behauptungen
> dieser Art nachzudenken.
> Nachdenken kann man dann, wenn ein Scheinprototyp dasteht. Das zu
> entlarven kann sogar Spass machen wenn es gut gemacht ist. Alles andere
> ist verlorene Zeit, weil die "Erfinder" eh keine Argumente zur Kenntnis
> nehmen.
Dem würde ich jetzt nicht so im Allgemeinen zustimmen. Bei irgendeinem 
Querkopf, der auf der Schiene ist 'Es funktioniert, und alle, die was 
anderes behaupten sind dumm' trifft das wahrscheinlich zu.
Aber es gibt auch die, die nicht wissen, warum es nicht funktioniert, 
aber es grundsätzlich wissen wollen. Bei denen ist es sehr viel 
sinnvoller es zu erklären als sie einfach als dumm hinzustellen.
Wer hat nicht als Kind oder Jugendlicher mal versucht, ein Perpetuum 
Mobile zu erfinden?
Allerdings sehe ich bei Magneten das Problem, dass die Erklärung halt 
nicht so intuitiv ist (wenn man dem Energieerhaltungssatz nicht glaubt).

von Dieter D. (Firma: Hobbytheoretiker) (dieter_1234)


Lesenswert?

Mark S. schrieb:
> Witzig bist Du also auch nicht.

Unwissen ist des Witzes sein Tod.

Monk schrieb:
> Niemand konnte bisher ein Perpetuum mobile bauen, ...

Von den Bierbrauern gab es ein Perpetuum Mobile zu bewundern. Das 
bekamen auch nur wenige zu sehen. Nach dem Gelage, wußte niemand mehr, 
wie es funktionierte. :(

von H.Joachim S. (crazyhorse)


Lesenswert?

Florian schrieb:
> Wer hat nicht als Kind oder Jugendlicher mal versucht, ein Perpetuum
> Mobile zu erfinden?

Ja, da war ich 10 und kam auf die geniale Idee den Motor meines 
Modellautos an den Dynamo des Fahrrads anzuschliessen :-)
Wer den Energieerhaltungssatz nicht akzeptieren will, muss als erstes 
zeigen dass es zumindest theoretisch die Möglichkeit gäbe diesen zu 
umgehen. Vorher ist Bastelei jeder Art sinnlos.

von Steven (muellermilch)


Lesenswert?

Ralf X. schrieb:
> Stell Dir einfach mal vor, Du lässt ein Rad oder Kugel ohne Luft- und
> Rollreibungsverlusten eine schräge Ebene von 45°
> (Beschleunigungsstrecke) runterrollen  und anschliessend eine 10°
> Steigung (Bremsstrecke) wieder nach oben bis auf die gleiche Höhe der
> Ausgangssituation.
> Deine Bremsstrecke ist also bedeutend länger, genauso die Zeit, die dort
> verbracht wird, aber das energetische Endergebnis ist null.
> Ebenso, wenn Du den Vorgang umdrehst.

Ah Denkfehler meinerseits. Der Bremsweg darf natürlich nicht länger 
sein. Im Gegenteil, er soll ja sogar kleiner sein im Vergleich zum 
Beschleunigungsweg. Es gibt ja einen Punkt im System, wo die 
Magnetkräfte aufhören zu wirken, sowohl auf der Beschleunigungseite als 
auch auf der Bremsseite. Diese Punkte sind entscheidend für die 
Bestimmung des Ungleichgewichts. Desweiteren behaupte ich, aufgrund der 
Ausrichtung der Magnete, dass die Energie auf der Beschleunigungsseite 
größer ist als auf der Bremsseite.
Ich mache mir mal Gedanken und versuche einen Prototyp zu bauen.
Dann werde ich ja vielleicht die pure Realität zu spüren bekommen und 
hoffentlich aus meiner Traumwelt aufmachen :D

von John B. (craftsman)


Lesenswert?

H.Joachim S. schrieb:
> Es lohnt sich einfach nicht über irgendwelche theoretischen Behauptungen
> dieser Art nachzudenken.

Ja, auf der inhaltlichen Ebene lohnt das nicht.

Auf der didaktischen Ebene lohnt es durchaus, zu überlegen, wie man 
Denkfehler aufzeigt, und wie man zu sinnvollen weiteren Überlegungen 
anregen kann. Es sollte einem aber dabei klar sein, dass man das 
liebgewonnene Wunschdenken, in das einer schon viel geistige Anstrengung 
hineingesteckt hat, nicht von jetzt auf gleich abstellen kann. Dazu 
braucht es einen geistigen Reifungsprozess, der Zeit braucht, so wie 
alle Reifungsprozesse durch die ein junger Mensch nach und nach in 
engeren Kontakt mit der Realität kommt. Die meisten schaffen das ja 
auch.

Um nochmals zum angeführten Zitat zurück zu kommen: "theoretische 
Behauptungen" habe ich hier nicht gesehen, eher kindliches Wunschdenken. 
In der Grundschule habe ich auch Perpetua mobile und alle möglichen 
mechanische Automaten erdacht. Das war sehr spannend, damals.

Steven schrieb:
> Am Ende muss ich doch nur wissen, ob die Bremsstrecke länger war als die
> Beschleunigungsstrecke.

Steven, du betrachtest jeweils nur Teilaspekte der Maschinerie und 
ziehst dann voreilige Schlüsse, die falsch sind. Wenn du dagegen alles 
einbeziehst, die Geometrie, die Wege, die Kräfte, dann kannst du die 
Energien ermitteln und blilanzieren. Und NUR auf die Energieblilanz 
kommt es an. Sie zeigt, was möglich ist, und was nicht.

Zur Zeit hast du zu wenig physikalisches Fachwissen und bist ungeübt 
darin, physikalische Überlegungen anzustellen. Wenn ich das Thema 
wirklich interessiert, dann lerne Physik. Aber bitte aus Lehrbüchern. So 
wie du versuchst im Forum zu diskutieren, vertiefst du nur deine 
Denkfehler.

PS:
Gerade sehe ich, dass du inzwischen noch etwas geschrieben hast. Darauf 
gehe ich jetzt nicht ein.

von H.Joachim S. (crazyhorse)


Lesenswert?

Warum?
Genausogut kann man sich auch mit Flacherdlern oer 
Reptiloidenverfechtern streiten.

von H.Joachim S. (crazyhorse)


Lesenswert?

Steven schrieb:
> Traumwelt aufmachen

Nicht aufmachen, aufwachen!

von Matthias S. (Firma: matzetronics) (mschoeldgen)


Lesenswert?

Monk schrieb:
> Ich kenne jemanden, der tausende Euro in die Idee versenkte,
> Benzin-Motoren mit Wasser effizienter zu machen.

Diese Experimente muss man gar nicht als Privatmann machen, denn VW hat 
vor vielen Jahrzehnten am klassischen Boxermotor den Zusammenhang 
zwischen Luftfeuchtigkeit und Leistungsabgabe gemessen und dazu eine 
Veröffentlichung geschrieben, die es in den VW Publikationen zu lesen 
gab. Es zeigte sich, wimre, das es eine Glockenkurve gab, bei der die 
Leistung des Motors tatsächlich anstieg bei erhöhter Luftfeuchtigkeit, 
dann aber wieder absank, wenn zu viel Feuchtigkeit die Verbrennung 
behinderte.
Damals wurde der Leistungsanstieg durch den zusätzlichen Wasserdampf 
erklärt.

Es ist lange her, seit ich das gelesen habe und deswegen habe ich die 
genauen Zahlen nicht mehr im Kopf. Die Steigerung der Leistung war aber 
so klein, das es sich zumindest für VW nicht lohnte, da mehr draus zu 
machen. Die Versuche wurden aber hauptsächlich zur Verifizierung des 
Motorlaufs in verschiedenen Klimata gemacht, der Motor sollte eben in 
den Alpen genauso gut laufen wie im Regenwald.

: Bearbeitet durch User
von Robert M. (r_mu)


Lesenswert?

Monk schrieb:
> Ich kenne jemanden, der tausende Euro in die Idee versenkte,
> Benzin-Motoren mit Wasser effizienter zu machen. Angeblich wollte er das
> einige male vor Publikum vorführen, wurde aber immer kurz vorher von
> höheren Mächten daran gehindert. Er gab die Idee letztendlich auf, als
> deutliche Anzeichen Verfolgungswahn aufkamen. Traurige Sache, aber
> leider ein typischer Fall.

Als Privatmensch ist man da auf verlorenem Posten, um sowas ernsthaft zu 
betreiben brauchts Prüfstände, Messgeräte, Motoren, ein Team und ein 
Budget. Die "low hanging fruit" bei Verbrennern sind alle längst 
abgeerntet.

von Timo W. (timo)


Lesenswert?

Monk schrieb:
> Ich kenne jemanden, der tausende Euro in die Idee versenkte,
> Benzin-Motoren mit Wasser effizienter zu machen.

hier schieb er von Effizienssteigerung


Matthias S. schrieb:
> bei der die
> Leistung des Motors tatsächlich anstieg bei erhöhter Luftfeuchtigkeit

... du hier von Leistungssteigerung.

Das sind zwei verschiedene Ziele.

Wassereinspritzung und Wasserdiesel sind bekannte funktionierende 
Verfahren zur Leistungssteigerung. Durch senkung der 
Brennraumtemperatur, senkung der Spitzendrücke und bessere Nutzung der 
Wärme.

Es kann zwar auch das Abgasverhalten verbessern und den Verbrauch 
minimal senken aber in erster Linie erhöht es die Leistung oder 
ermöglicht stärkeres downsizing.

Auf der anderen Seite gibt es aber problematische Verschwörer die 
predigen es wäre so möglich den großteil des Treibstoffs zu ersetzen. Da 
werden dann gerne mal einsparungen von 50%-80% versprochen. Man braucht 
nur noch ein wenig Geld um es Serienreif zu entwickeln ...

von Matthias S. (Firma: matzetronics) (mschoeldgen)


Lesenswert?

Timo W. schrieb:
> Das sind zwei verschiedene Ziele.

Wollen wir mal keine Haare spalten. Natürlich wurde bei VW die gleiche 
Menge Kraftstoff auf dem Prüfstand verwendet, nur bei unterschiedlichen 
Luftfeuchtigkeiten. Die Leistungssteigerung war also eine Steigerung der 
Effizienz.

von Timo W. (timo)


Lesenswert?

Steven schrieb:
> Desweiteren behaupte ich, aufgrund der
> Ausrichtung der Magnete, dass die Energie auf der Beschleunigungsseite
> größer ist als auf der Bremsseite.
> Ich mache mir mal Gedanken und versuche einen Prototyp zu bauen.
> Dann werde ich ja vielleicht die pure Realität zu spüren bekommen und
> hoffentlich aus meiner Traumwelt aufmachen :D

Stell dir vor du lässt eine Kugel einen Hang runterrollen und auf der 
anderen Seite wieder rauf. Alle Beobachtungen (Physikalische regeln) 
zeigen das die Kugel am Ende nie höher rollen wird als sie an Anfang 
war. (Ohne weiteren Antrieb und ruhend am Anfang). Am Ended wird die 
Kugel unten im Tal liegen.

Du behauptest sinngemäß das du durch veränderung des Geländeprofils 
erreichst das die Kugel am Ende höher ist. Und dann könntests due sie 
oben zurück rollen lassen und sie würde ewig immer schneller durchs Tal 
rollen. Egal wie du das Gelände änderst oder Wippen einbaust die Kugel 
wird nie höher rollen als am Anfang. Probiers aus.

Dein "Magnetmotor" verhält sich genau so. Alle Ingenieuere und 
Naturwissenschaftler die mit Magnetfeldern arbeiten/rechnen wissen das 
man am Ende wieder auf dem gleichen Potential im Magnetfeld enden muss. 
Egal wie wirr der weg auch wird am Ende ist die Energie null.

Physikalische gesetze haben nichts mit Jura zu tun. Newton war kein 
Jurist. Der hat nicht eines Tages gesagt ich erlasse jetzt die 
Newton'schen Gesetzte ab Morgen haben sich alle Objekte dran zu halten. 
Er hat die Umwelt beobachtet und beschrieben wie sie sich verhält. Das 
ist Grundsätzlich so bei Naturwissenschaftlichen "Gesetzen": Das sind 
Beschreibungen des Verhaltens die über Jahrhunderte gemacht und 
verifiziert wurden und nicht willkürlich erlassene Regeln. Deshalb kann 
man diese Gesetze auch nicht einfach ändern auch wenn so mancher das 
gerne tun würde

von Timo W. (timo)


Lesenswert?

Matthias S. schrieb:
> Wollen wir mal keine Haare spalten. Natürlich wurde bei VW die gleiche
> Menge Kraftstoff auf dem Prüfstand verwendet, nur bei unterschiedlichen
> Luftfeuchtigkeiten. Die Leistungssteigerung war also eine Steigerung der
> Effizienz.

Das ging aus deinen Ausführungen nicht hervor. Durch die feuchtigkeit 
ändert sich der Arbeitspunkt und für mich ist es eher ungewöhnlich bei 
Performancemessungen ausgerechnet die Kraftstoffmenge als sollwert zu 
verwenden. Hast du zufällig einen Link zu der Veröffentlichung?

von Steven (muellermilch)


Lesenswert?

Timo W. schrieb:
> Du behauptest sinngemäß das du durch veränderung des Geländeprofils
> erreichst das die Kugel am Ende höher ist. Und dann könntests due sie
> oben zurück rollen lassen und sie würde ewig immer schneller durchs Tal
> rollen. Egal wie du das Gelände änderst oder Wippen einbaust die Kugel
> wird nie höher rollen als am Anfang. Probiers aus.

Sinngemäß erhöhe ich das Gewicht der Kugel zwischen Startpunkt und Tal 
und verringere dann wieder das Gewicht der Kugel zwischen Tal und 
Endpunkt.
Ich verstehe nicht, warum das Kräfteungleichgewicht der Magente 
ignoriert wird. Es wirken doch ganz klar unterschiedliche Kräfte 
aufgrund der Ausrichtung der Magnete zwischen beiden Seiten. Schräg 
ausgerichtete Magnete haben doch nicht die gleichen Kräfte wie frontal 
ausgerichtete Magnete, oder etwa doch?

: Bearbeitet durch User
von Steven (muellermilch)


Lesenswert?

Steven schrieb:
> Ich verstehe nicht, warum das Kräfteungleichgewicht der Magente
> ignoriert wird. Es wirken doch ganz klar unterschiedliche Kräfte
> aufgrund der Ausrichtung der Magnete zwischen beiden Seiten. Schräg
> ausgerichtete Magnete haben doch nicht die gleichen Kräfte wie frontal
> ausgerichtete Magnete, oder etwa doch?

Natürlich weiß ich nicht, wie sich diese Kräfte auf den Rotor auswirken, 
da die wirkende Kräfte der Magnete immer einen anderen Winkel haben und 
dadurch sich der Drehmoment ständig verändert. Aber genau das weiß ich 
ja nicht, ohne es zu berechnen oder zu testen.

von Alexander (alecxs)


Lesenswert?

Giovanni schrieb:
> Lieber Steven,
> Auch für Dich gilt:
>
> der Energieerhaltungssatz

Wo siehst Du den verletzt?

Beitrag "Re: Motor, der Magnetenergie verbraucht - möglich?"

von Ralf X. (ralf0815)


Lesenswert?

Alexander schrieb:
> Giovanni schrieb:
>> Lieber Steven,
>> Auch für Dich gilt:
>>
>> der Energieerhaltungssatz
>
> Wo siehst Du den verletzt?
>
> Beitrag "Re: Motor, der Magnetenergie verbraucht - möglich?"

Der nächste Fantast...

von Alexander (alecxs)


Lesenswert?

erst denken, dann reden..

von Matthias S. (Firma: matzetronics) (mschoeldgen)


Lesenswert?

Timo W. schrieb:
> Hast du zufällig einen Link zu der Veröffentlichung?

Das war lange vor der Zeit des Internet und muss so Ende 60er, Anfang 
70er Jahren entstanden sein, als VW noch alle Kohle mit dem Boxermotor 
verdient hat. Link ist also nicht.
Wir haben uns damals (Ende der 70er) mal aus Interesse technische 
Unterlagen zum 1300ccm Motor kommen lassen, weil das das Auto war, mit 
dem wir dann tausende von Km durch Europa gefahren sind. Darin waren 
eben auch die Grafiken zum Verbrauch und die Abhängigkeit von Luftdruck, 
Luftfeuchte und viele andere Dinge. VW hat alles an diesem Motor 
gemessen, was zu messen war.

von Ralf X. (ralf0815)


Lesenswert?

Matthias S. schrieb:
> Timo W. schrieb:
>> Hast du zufällig einen Link zu der Veröffentlichung?
>
> Das war lange vor der Zeit des Internet und muss so Ende 60er, Anfang
> 70er Jahren entstanden sein, als VW noch alle Kohle mit dem Boxermotor
> verdient hat. Link ist also nicht.
> Wir haben uns damals (Ende der 70er) mal aus Interesse technische
> Unterlagen zum 1300ccm Motor kommen lassen, weil das das Auto war, mit
> dem wir dann tausende von Km durch Europa gefahren sind. Darin waren
> eben auch die Grafiken zum Verbrauch und die Abhängigkeit von Luftdruck,
> Luftfeuchte und viele andere Dinge. VW hat alles an diesem Motor
> gemessen, was zu messen war.

Zeit meines Lebens habe ich immer wieder etwas von der 
Wassereinspritzung aus den unterschiedlichsten Ecken gelesen, gehört 
oder gesehen.
VW war nur einer von vielen, die sich damit beständig beschäftigt haben.
Einen kleinen Einblick findet man auch über Wiki, bzw. die dortigen 
Quellen:

https://de.wikipedia.org/wiki/Wassereinspritzung

von Alexander (alecxs)


Lesenswert?

Finger weg davon, du wirst vom Staat gefickt. Horst Kirsten, GFE
http://www.gfe-skandal.de

oder Erste Mai GmbH

Funktioniert aber wirklich

https://schlattmann.de/forum/viewtopic.php?t=661&start=10

: Bearbeitet durch User
von Florian (flori_n)


Lesenswert?

Steven schrieb:
> Ich verstehe nicht, warum das Kräfteungleichgewicht der Magente
> ignoriert wird.
Selbst wenn es so ist, musst du wieder Energie 'verbrauchen', um den 
Magnet zu drehen. Einerseits wegen der Reibung im Lager, andererseits 
aber auch, weil der Magnet anstößt und dadurch gebremst wird.
Um den Magnet in Drehung zu versetzen, musst du Energie in den Magnet 
stecken. Die Energie wird dann 'vernichtet', wenn der Magnet am Anschlag 
gebremst wird. Diese Energie ist dann für das Rad verloren.

Timo W. schrieb:
> Du behauptest sinngemäß das du durch veränderung des Geländeprofils
> erreichst das die Kugel am Ende höher ist.
Da ist mir wieder der passender Xkcd-Comic eingefallen: 
https://xkcd.com/2217/

Wenn jemand behauptet, er könnte aus 52 Spielkarten nur durch Mischen 
und ohne Tricks 53 Spielkarten machen, muss man nicht erst den Fehler 
suchen. Es geht halt nicht, egal wie gut oder kompliziert man mischt. 
(1)
Man kann jetzt sagen, dass sich Karten nicht einfach vermehren können, 
das ist jedem klar und entspricht beim scheinbaren Perpetuum Mobile dem 
Energieerhaltungssatz. Und trotzdem kommst du und fragst, übertragen 
gesagt, warum dein Kartenmischverfahren nicht funktioniert, um aus 52 
Karten 53 Karten zu machen.

Jetzt behaupte ich mal, es gibt ein Mischverfahren, mit dem man aus 52 
Karten ohne Hinzufügen einer neuen Karte 53 Karten machen kann. Die 
Erklärung, dass Karten sich ohne Zufuhr von außen nicht vermehren 
können, lasse ich nicht gelten. Jetzt erklär mir mal, warum ich falsch 
liege. Das ist gar nicht so einfach.

(1) Kann man den Satz von Banach-Tarski auch auf Energie übertragen? 
Vielleicht ist das der Ansatz für ein Perpetuum Mobile... ;-)

von Wolfgang R. (Firma: www.wolfgangrobel.de) (mikemcbike)


Lesenswert?

Jatzt kommt das Schwurblertreffen richtig in Gang... Die da Oben wollen 
alles verhindern... Auch Magnetmotoren.

von Timo W. (timo)


Lesenswert?

Alexander schrieb:
> Funktioniert aber wirklich

Es funktioniert nicht wie gepredigt. Wassereinspritzung erlaubt mehr 
downsizing, Leistungssteigerung und besseres Abgasverhalten. 
Kraftstoffeinsparungen sind auch möglich aber minimal. Einsparungen von 
wenigen Prozent sind möglich und Abgasnachbehandlung wird einfacher. Es 
rechtfertigt aber meistens den Aufwand nicht.

Gepredigt wurde von diesen Betrügern aber erhebliche Einsparungen im 
hohen zweistelligen Bereich. Für die Entwicklung wurden vorrauszahlungen 
genommen und davon gelebt und die vorherigen Kunden subventioniert. Das 
war ganz einfach ein Schneballsystem. Das ist volkommen zu Recht Betrug.

Leichtgläubige Menschen wollen aber lieber dem Betrüger glauben.

von Wolfgang R. (Firma: www.wolfgangrobel.de) (mikemcbike)


Lesenswert?

Alexander schrieb:
> Finger weg davon, du wirst vom Staat gefickt.

Du hast vergessen zu erwähnen, dass die Betrügerbande rechtskräftig zu 
mehreren Jahren gefängnis verurteilt wurde, weil sie viele gutgläubige 
Anleger mit Lug und Trug um ihr Geld gebracht haben.

Wer hier wen gefickt hat, ist wohl mittlerweile eindeutig.

von Alexander (alecxs)


Lesenswert?

Klar wenn man containerweise BHKW in Millionenhöhe beschlagnahmt und die 
Firma weder liefern noch die in den Gütern steckenden Vorauszahlungen 
zurück zahlen kann, dann ist das auf jeden Fall ein Schneeballsystem. Da 
hat die Staatsanwaltschaft dafür gesorgt.

von Timo W. (timo)


Lesenswert?

Steven schrieb:
> Sinngemäß erhöhe ich das Gewicht der Kugel zwischen Startpunkt und Tal
> und verringere dann wieder das Gewicht der Kugel zwischen Tal und
> Endpunkt.

Nein das tust du nicht. Und selbst wenn wäre das Ergebnis gleich 
abgesehen von dem energetischem Aufwand das gewicht zu Ändern.

Steven schrieb:
> Ich verstehe nicht, warum das Kräfteungleichgewicht der Magente
> ignoriert wird. Es wirken doch ganz klar unterschiedliche Kräfte
> aufgrund der Ausrichtung der Magnete zwischen beiden Seiten. Schräg
> ausgerichtete Magnete haben doch nicht die gleichen Kräfte wie frontal
> ausgerichtete Magnete, oder etwa doch?

Es wird nichts ignoriert. Du willst es nur nicht sehen. Die Felder der 
Magnete ändern sich nicht. Du glaubst nur durch ein kompliziertes 
Bewegungsmuster der Magnete umeinander die konservativen eigenschaften 
des Feldes zu umgehen. Egal wie du den einen Magneten durch das Feld des 
anderen bewegst wenn du wieder auf die selbe Position kommst hast du 
genau so viel in die Trennung voneinander gasteckt wie du beim anziehen 
gewonnen hast.
Deshalb das Beispiel mit der Kugel im Gravitationsfeld: Egal wie komplex 
du versuchst die Kugel durch das Feld zu bewegen wirst du nie ein 
höheres Potential erreichen als zu beginn (wie immer ohne weitere 
Energie rein zu stecken)

von Uwe S. (bullshit-bingo)


Lesenswert?

Steven schrieb:
> Ich verstehe nicht, warum das Kräfteungleichgewicht der Magente
> ignoriert wird.

Es gibt keines. Habe es oben erklärt, du musst es nur noch verstehen.
Schau dir an, in welche Richtung wann welche Kraft tatsächlich auf die 
Drehbewegung des Rotors wirkt. Und vergleiche beides für das 
Beschleunigen und das Bremsen.
Beim Beschleunigen hast du für längere Zeit entweder niedrige Kraft bei 
geeignetem Winkel, oder hohe Kraft bei ungeeignetem Winkel. Nach dem 
Anschlag der Magnete hast du für kurze Zeit eine hohe Kraft rückwärts im 
perfekten Winkel.
Durch diese Magnet-Lagerungs- und Anschlaggeschichte hast du sogar einen 
"Motor" gebaut, der nicht mal so oft hin und her schwingt, wie einer nur 
mit zwei gewöhnlichen, starren Magneten. Denn es wird in deinem Fall 
noch zusätzlich Reibung erzeugt. So schlecht ist dieses scheinbar tolle 
Konstrukt. Akzeptieren musst du es, dann sparst du dir jede Menge 
sinnlose Arbeit.

von Wolfgang R. (Firma: www.wolfgangrobel.de) (mikemcbike)


Lesenswert?

Alexander schrieb:
> Klar

Zitat aus den Stuttgarter Nachrichten von 2014:

"Es hörte sich ja auch verlockend und plausibel an. Die modernen BHKW 
sollten in Altenheimen, Kliniken, großen Wohnanlagen und Schulen jeweils 
7500 Stunden pro Jahr laufen .Weil man Großabnehmerrabatte bekomme, 
staatliche Förderungen optimal ausschöpfe und die Umsatzsteuer 
rückerstattet werde, seien hohe Renditen drin. Zwar bestellte die Erste 
Mai tatsächlich mehrere BHKW. Sie hatte jedoch nicht einen einzigen 
Stellplatz, es ging kein einziges Kraftwerk je in Betrieb."

und:

"Nachdem die Angeklagte mit 190 000 Euro in der Tasche die Firma 
verlassen hatte, warben die drei Männer weitere vier Millionen Euro von 
weiteren 94 Anlegern ein.

Von den insgesamt acht Millionen Euro ist das meiste Geld weg. Die 
Betrüger genehmigten sich großzügige Gewinnentnahmen und Provisionen. 
Einer der Herren gewährte seiner Gattin ein Darlehen von 1,5 Millionen 
Euro aus Firmengeldern, mit dem sie eine Villa in bester Stuttgarter 
Lage finanzierte.

Die drei betrügerischen Geschäftsmänner waren bereits von einer anderen 
Strafkammer zu wenig luxuriösen Gefängnisaufenthalten von fünfeinhalb 
bis zu knapp sieben Jahren verurteilt worden. Die Frau als Nachzüglerin 
kam jetzt wegen ihres Geständnisses und einer Schadenswiedergutmachung 
in Höhe von 100 000 Euro mit Bewährung davon. Das war nicht unbedingt zu 
erwarten gewesen. Schließlich brachte die Ingenieurin bereits eine 
Bewährungsstrafe mit in den Prozess. Vor Jahren hatte sie für eine 
Anlagefirma gearbeitet, in der auch einer ihrer BHKW-Komplizen sein 
Unwesen getrieben hatte. Diese Firma hatte fast 1600 Anleger um sage und 
schreibe 16 Millionen Euro betrogen."

https://www.stuttgarter-nachrichten.de/inhalt.millionenbetrug-kleinanleger-um-altersvorsorge-gebracht.31a0aa09-d600-475c-a494-26fc5bd38440.html

Betreibe hier mal bitte keine Täter-Opfer-Umkehr!

von Timo W. (timo)


Lesenswert?

Alexander schrieb:
> Klar wenn man containerweise BHKW in Millionenhöhe beschlagnahmt und die
> Firma weder liefern noch die in den Gütern steckenden Vorauszahlungen
> zurück zahlen kann, dann ist das auf jeden Fall ein Schneeballsystem. Da
> hat die Staatsanwaltschaft dafür gesorgt.

Das ist eine freche Lüge. Die haben nie auch nur ein einziges ihrer 
Wunderkraftwerke geliefert. Die haben stinknormale BHKW verkauft und mit 
den Anzahlungen der Neukunden die Kraftstoffe der Altkunden finanziert. 
Das ist schlicht Betrug.

von Alexander (alecxs)


Lesenswert?

Freche Lüge ist zu behaupten die Beschlagnahmung hätte nicht die Firma 
ruiniert.

Ich habe die Briefe aus der JVA gelesen. In meinen Augen ist Horst 
Kirsten ein ehrlicher Ingenieur der an seine Sache glaubt, und kein 
schlitzohriger Unternehmer der mit dubiosen Hintermännern das ganz große 
Geld scheffeln wollte. Du kannst Du gerne das Psiram Märchenbuch 
zuklappen, mein Urteil in der Sache steht fest.

von Wolfgang R. (Firma: www.wolfgangrobel.de) (mikemcbike)


Lesenswert?

Alexander schrieb:
> Du kannst Du gerne das Psiram Märchenbuch
> zuklappen, mein Urteil in der Sache steht fest.

Und die Erste Mai GmbH wollte auch nur unser Bestes. Unser Geld.

Ach, und wo du es oben erwähnt hast: hier kann man sich selbst ein Bild 
machen - genügend Quellenangaben stehen ja drin:

https://www.psiram.com/de/index.php/GFE-Skandal

Und um gleich mal wieder On-Topic zu kommen - Märchenaus dem 
Magnetmotorwald:

https://www.psiram.com/de/index.php/Magnetmotor

: Bearbeitet durch User
von Alexander (alecxs)


Lesenswert?

Wolfgang R. schrieb:
> Und die Erste Mai GmbH wollte auch nur unser Bestes. Unser Geld.

Nein, die wollten mit einer kommissarischen Revolutionsregierung das 
Proletariat vom Kapitalismus befreien!

Ich habe aber von Horst Kirsten gesprochen, hat mit denen gar nichts zu 
tun.

von Timo W. (timo)


Lesenswert?

Alexander schrieb:
> Freche Lüge ist zu behaupten die Beschlagnahmung hätte nicht die Firma
> ruiniert.
>
> Ich habe die Briefe aus der JVA gelesen. In meinen Augen ist Horst
> Kirsten ein ehrlicher Ingenieur der an seine Sache glaubt, und kein
> schlitzohriger Unternehmer der mit dubiosen Hintermännern das ganz große
> Geld scheffeln wollte. Du kannst Du gerne das Psiram Märchenbuch
> zuklappen, mein Urteil in der Sache steht fest.

Ob er dran glaubt, ob du dran glaubst, ist vollkommen egal. Die technik 
die er angeblich verkauft hat existierte nie.
Wenn ein Unternehmer Anzahlungen/Vorrauszahlungen nimmt gehört ihm 
dieses Geld nicht. Anzahlungen sind kein Gewinn. Diese Anzahlungen 
anderweitig zu verwenden war veruntreuung bzw mit diesem System schlicht 
Betrug.

von Wolfgang R. (Firma: www.wolfgangrobel.de) (mikemcbike)


Lesenswert?

Alexander schrieb:
> Ich habe aber von Horst Kirsten gesprochen, hat mit denen gar nichts zu
> tun.

Du hast aber alle in einem Aufwasch genannt. Dass Kirsten in einen 
anderen Betrugsfall verwickelt war, ist klar.

von Robert M. (r_mu)


Lesenswert?

Timo W. schrieb:
> Es wird nichts ignoriert. Du willst es nur nicht sehen. Die Felder der
> Magnete ändern sich nicht. Du glaubst nur durch ein kompliziertes
> Bewegungsmuster der Magnete umeinander die konservativen eigenschaften
> des Feldes zu umgehen. Egal wie du den einen Magneten durch das Feld des
> anderen bewegst wenn du wieder auf die selbe Position kommst hast du
> genau so viel in die Trennung voneinander gasteckt wie du beim anziehen
> gewonnen hast.

Fairerweise muss man dazusagen dass das Magnetfeld an und für sich nicht 
konservativ ist. Vielleicht kommt daher die Idee dass sich da was 
"rausholen" lässt. Ist natürlich Blödsinn. Wenn man das System ein wenig 
idealisiert ist es sehr wohl konservativ, ohne die Idealisierung handelt 
man sich nur Verluste ein (Reibung, EM-Wellen, 
Magnetisierung/Verschiebung, joul'sches Heizen etc...)

"Zu Fuss" zu Bewegungsgleichungen zu kommen sieht nach einem äußerst 
komplizierten Unterfangen aus, selbst wenn man viel idealisiert und 
alles "statisch" behandelt.

Und es hilft einem auch nichts das auf diese Art zu analysieren oder zu 
"verstehen" wie oben schon geschrieben. Wie man es auch dreht und 
wendet, der Formalismus der Elektrodynamik hat die Energieerhaltung 
eingebaut, davon kann man also schon aus Prinzip nichts ableiten wo 
etwas anderes herauskommt.

von Tilo R. (joey5337) Benutzerseite


Lesenswert?

Steven schrieb:
> Meine Beschleunigungsseite ist also größer als
> meine Bremsseite. Ob allerdings nun dieses Ungleichgewicht ausreicht,
> sowohl den Reibungsverlust auszugleichen und genügend Überschuss übrig
> zu haben, gilt es zu klären und herauszufinden.

Es geht nicht nur im die Strecke, sondern auch um die auf dieser Strecke 
wirkende Kraft.
Ganz grundsätzlich gilt für die Energie = Kraft x Weg.
Natürlich ändert sich die Kraft unterwegs, die Kraft wird damit abhängig 
vom Ort (ich nenne den jetzt mal x):

Das ist die Energie, wenn man die Magnete von einem Ort/Abstand X1 zum 
Ort/Abstand X2 verändert. Und natürlich zählt nur die Kraft in/gegen die 
Bewegungsrichtung, korrekterweise müsste man das also vektoriell, als 
Linienintegral schreiben.
Wirkt die Kraft in Bewegungsrichtung bekommst du Energie raus, wirkt sie 
gegen die Bewegungsrichtung wird das Ergebnis negativ, du musst Energie 
aufwenden.
Dummerweise ist das Magnetfeld ein Potentialfeld. Es ist daher völlig 
egal, auf welchem Weg du von X1 zu X2 läufst oder welche 
Zwischenstationen man macht. Die Kraft kann kleiner sein, dann ist der 
Weg länger, oder andersrum. Genauso kann die Kraft zeitweise mit, gegen 
oder schräg zur Bewegungsrichtung gehen - das spielt keine Rolle.
Am Ende hängt die Energie nur vom Abstand am Anfang und am Ende ab.

Wenn sich die Magneten jetzt ausrichten, so kommt die Energie aus diesem 
Potentialfeld. Am Ende sind die Magneten näher bei einander, d.h. an 
einem Ort mit noch weniger Energie. Und genau diese Energie musst du 
zusätzlich aufwenden, um sie wieder auseinander zu bringen.

Auch wenn man diese Magnetfeld irgendwie abschirmen wollte, so muss man 
die Abschirmung in den Spalt schieben, was wieder Energie kostet.

Physik ist Mist, nicht mal das einfachste Perpetuum Mobile funktioniert!

von Dieter D. (Firma: Hobbytheoretiker) (dieter_1234)


Lesenswert?

H.Joachim S. schrieb:
> Flacherdlern

Das sind die, die im Flachen Land zwischen Meer und Alpen Wohnen.

> Reptiloidenverfechtern

Die Reptiloiden in der Spielwarenabteilung mit Fernsteuerung in USA 
waren toll, aber das war leider vor den ersten Smartphones mit Kamera. 
:(

Daher gab es kaum Videos. Seit Corona spuckt die Suchmaschine immer 
diesen mystischen Reptiloidenkäse bestimmter Gruppen aus.

von Steven (muellermilch)


Angehängte Dateien:

Lesenswert?

Danke an diejenigen die versuchen es mir physikalisch zu erklären, 
leider verstehe ich es aber immernoch nicht. Um es mir vielleicht besser 
zu erklären, habe ich deswegen ein einfaches Beispiel gebaut. Aufgeteilt 
in zwei Gif's. Beachtet bitte, dass ihr den Namen des Gif's anklicken 
müssen, damit die Animation läuft, alos nicht das Bild.

Gif 1 - Kugelbahn statisch:

Das erste Gif zeigt eine stinknormale Kugelbahn. Das Prinzip ist euch ja 
bekannt. Betrachtet den Magneten bitte also statisch, ich habe vergessen 
diesen optisch zu fixieren.
Wir schauen uns hier nur die Beschleunigsseite an und vergleichen diese 
mit dem Gif 2, die Bremsseite wird ignoriert.

Gif 2 - Kugelbahn dynamisch:

Hier ist nun der Magnet dynamisch und er passt sich immer der Kugel 
peferkt an. Auch hier betrachten wir nur die Beschleunigungsseite und 
vergleichen diese mit Gif 1.

Wie sieht denn aber nun der Unterschied aus zwischen Gif 1 und Gif 2? 
Gibt es unterschiedliche Geschwindigkeiten auf der 
Beschleunigungsstrecke oder ist diese in beiden Fällen identisch? Und 
wenn ja, warum ist das so?

von Uwe S. (bullshit-bingo)


Lesenswert?

Steven schrieb:
> Gibt es unterschiedliche Geschwindigkeiten auf der
> Beschleunigungsstrecke oder ist diese in beiden Fällen identisch?

Es gibt Unterschiede, aber sie liegen genau nicht so, wie du sie gern 
hättest. Und zwar wird die max. Geschwindigkeit der Kugel beim drehbaren 
Magneten sogar etwas geringer sein. Weil am Drehpunkt des Magneten 
Reibung entsteht.
Warum die Ausrichtung des Magneten zur Kugel hin rein gar nichts bringt, 
habe ich inzwischen 2x erklärt...

Wenn man diese neue Simulation noch zu Ende betrachtet, also den 
Magneten plötzlich stoppt, so wird die Kugel noch schneller als erwartet 
gestoppt, weil im Anschlagmoment auch noch die kin. Energie des Magnets 
verloren geht. Wie gesagt, das Konstrukt bremst die Kugel sogar stärker 
als ein starr montierter Magnet. Über dem starren Magneten wird die 
Kugel mehrfach hin und her rollen, beim drehbaren Magneten wird dieses 
Pendeln schneller abnehmen.
Um das zu verstehen, musst du die Kräfte nebst ihren Vektoren 
einzeichnen, daraus die resultierende Kraft für den Vortrieb der Kugel 
ableiten.
Beim starren Magneten hast du zwar einen kürzeren Weg zum Anziehen der 
Kugel, die Kraft ist aber erheblich größer, und die Richtung stimmt 
auch. Unterm Strich führen beide Konstrukte zur selben max. 
Geschwindigkeit, bis auf die mech. Verluste des Magneten selbst.

von Carypt C. (carypt)


Lesenswert?

ich meine auf youtube ca 2005 schon mal eine rotierende Messingscheibe 
mit vielen angebrachten Mageten und einem dagegen gehaltenen 
Festmagnetenhalter gesehen zu haben, vielleicht hatte man kurz 
angeschubst, danach nahm das Ding schwer an Fahrt auf und blieb dann so. 
Es waren Magneten im Winkel angebracht um Anziehung und Abstoßung zu 
optimieren. Klar könnte man sagen, Druckluft, aber falls nicht, dann hat 
man trotzdem gesagt: es funktioniert nicht ; obwohl es dreht. Aber nicht 
weil es nicht drehte, sondern weil es kein Perpetuum Mobile sei, weil 
Magnete in 100 Jahren 5% schwächer werden, aber es dreht und also 
funktioniert es. Nur es ist kein Perpetuum Mobile.

(Hat das Schwächerwerden der Magneten etwas mit Heisenberg zu tun ?)

von Wolfgang R. (Firma: www.wolfgangrobel.de) (mikemcbike)


Lesenswert?

Woher kommt eigentlich die Energie, die nötig ist, den Magneten zu 
bewegen?

Noch mal: es wird nicht funktionieren.

Carypt C. schrieb:
> ich meine auf youtube ca 2005 schon mal eine rotierende Messingscheibe
> mit vielen angebrachten Mageten und einem dagegen gehaltenen
> Festmagnetenhalter gesehen zu haben

Aha...

Carypt C. schrieb:
> aber es dreht und also
> funktioniert es.

Sicher nicht. Nur so lange, bis die Reibung alle Energie in Wärme 
umgewandelt hat. Dafür brauche ich kein Magnetgedöhns, da reicht auch 
ein einfaches rotierendes Rad.

von Peter N. (alv)


Lesenswert?

Christian B. schrieb:
> BTW: Man kann Magnete nicht in Nord- und Südpol aufteilen und getrennt
> voneinander einbauen.

Und wie ist das bei magnetischen Monopolen?  :)

von Wolfgang R. (Firma: www.wolfgangrobel.de) (mikemcbike)


Lesenswert?

Peter N. schrieb:
> Und wie ist das bei magnetischen Monopolen?

Genau wie wirtschaftliche Monopole. Nicht gerne gesehen.

von ●DesIntegrator ●. (Firma: FULL PALATINSK) (desinfector) Benutzerseite


Lesenswert?

passiert hier noch irgendwas wichtiges?

von Wolfgang R. (Firma: www.wolfgangrobel.de) (mikemcbike)


Lesenswert?

●DesIntegrator ●. schrieb:
> passiert hier noch irgendwas wichtiges?

Das kann nur Steven beantworten - ich denke, er wird jetzt ein 
Funktionsmuster aufbauen und sich nach Auswertung der Ergebnisse nicht 
mehr weiter melden. ;-)

von Peter N. (alv)


Lesenswert?

Christian B. schrieb:
> BTW: Man kann Magnete nicht in Nord- und Südpol aufteilen und getrennt
> voneinander einbauen.

Und wie ist das bei magnetischen Monopolen?  :)

Steven schrieb:
> Kennt ihr die Abschussrampe, wo eine Kugel auf einer Magnetbahn rollt?

Bau doch mal eine ringförmige Bahn, bei der die Magnete die Metallkugel 
im Keis beschleunigt.
Das dürfte erstmal einfacher sein, als der Magnetmotor.

von Alexander (alecxs)


Lesenswert?

Carypt C. schrieb:
> Aber nicht weil es nicht drehte, sondern weil es kein Perpetuum Mobile
> sei,

Das ist ja der Witz an der Sache, es wird überhaupt kein 
Energieerhaltungssatz verletzt. Natürlich drehen Magnetmotoren. Nur sind 
die so unwirtschaftlich dass die keiner baut.

Carypt C. schrieb:
> weil Magnete in 100 Jahren 5% schwächer werden

Das geht wesentlich schneller. Das kannst Du schon an deinen 
Kühlschrankmagneten beobachten.

von Robert M. (r_mu)


Lesenswert?

Tilo R. schrieb:
> Es geht nicht nur im die Strecke, sondern auch um die auf dieser Strecke
> wirkende Kraft.

[...]

Sorry, aber der Post sieht schlecht GPT-halluziniert aus. Die Notation 
beim Integral macht mmn keinen Sinn (nach d/dx integrieren????). Das 
Magnetfeld ist außerdem eben kein Potentialfeld. Im Allgemeinen ist es 
ist nicht völlig egal wie man von x1 nach x2 kommt (man denke an einen 
stromdurchflossenen Leiter den man unterschiedlich oft umkreist).

von Wolfgang R. (Firma: www.wolfgangrobel.de) (mikemcbike)


Lesenswert?

Alexander schrieb:
> Natürlich drehen Magnetmotoren. Nur sind
> die so unwirtschaftlich dass die keiner baut.

Die sind nicht nur unwirtschaftlich, die sind sogar unsinnig. Wie gesagt 
- ohne Magnete würden die auch drehen, bis die Reibung die 
Rotationsenergie gefressen hat.

von Robert M. (r_mu)


Lesenswert?

Ein anderer Vergleich, der noch nicht gekommen ist: so ein "Magnetmotor" 
ist eine Art große drehende Schwungmasse, eine Schwungmasse dreht sich 
auch weiter wenn entsprechend angestossen, auch recht lange wenn passend 
gelagert.

Hier gibts übrigens ein "echtes" perpetuum mobile zu sehen 
https://www.technischesmuseum.at/tmw-zine/perpetuum_mobile, bei dem 
Gerät war lange unklar wie es funktioniert und ich bin nicht sicher ob 
das inzwischen aufgeklärt ist, kenn den podcast noch nicht.

: Bearbeitet durch User
von Marcel V. (mavin)


Angehängte Dateien:

Lesenswert?

Peter N. schrieb:
> Bau doch mal eine ringförmige Bahn, bei der die Magnete die Metallkugel
> im Keis beschleunigt.
> Das dürfte erstmal einfacher sein, als der Magnetmotor.

Die Kugel würde dann aber vorne am Ende der Magnetbahn abrupt stehen 
bleiben. Trotzdem könnte man sich diesen Effekt zu Nutze machen, um eine 
zweite Kugel, die bereits vorne in Startposotion liegt, anzustoßen.

von Uwe S. (bullshit-bingo)


Lesenswert?

Ich verstehe nicht, warum Magnetmotoren ständig mit Perpetua Mobilia 
verglichen werden. Letztere sind ja sogar noch denkbar, widersprächen 
keinem phys. Gesetz, lediglich gemachten Erkenntnissen. Aber Energie aus 
dem Nichts zu gewinnen ist die totale Lachnummer, ganz klar in 
Lummerland beheimatet...

von Dieter D. (Firma: Hobbytheoretiker) (dieter_1234)


Lesenswert?

Der TO sollte auch an den Energieaifwand für und durch die 
Magnetorestriktion denken.

von Marcel V. (mavin)


Angehängte Dateien:

Lesenswert?

Marcel V. schrieb:
> ...um eine zweite Kugel, die bereits vorne in der Startposition liegt,
> anzustoßen.

Genau das wurde hier tatsächlich so gemacht. Im Video hat man nur 
versäumt, die zweite Kugel über eine Kreisbahn zurück zur Startposition 
zu führen, damit das System selbständig am "Leben" erhalten bleibt.

Die Impulskraft ist ziemlich enorm. Eine Rückführung der zweiten Kugel 
an die Startposition, ist dadurch vom Energiegehalt her, also problemlos 
machbar.

von Alexander (alecxs)


Lesenswert?

Uwe S. schrieb:
> Aber Energie aus dem Nichts zu gewinnen ist die totale Lachnummer, ganz
> klar in Lummerland beheimatet...

Ja. oder Energie aus Licht. was für ne Esoteriknummer.

von Wolfgang R. (Firma: www.wolfgangrobel.de) (mikemcbike)


Lesenswert?

Alexander schrieb:
> Ja. oder Energie aus Licht. was für ne Esoteriknummer.

Das zwischen Licht und Nichts noch ein Unterschied besteht, ist dir aber 
schon klar? Auch wenn es ähnlich klingt...

Versuchst du eigentlich, hier mühsam witzig zu sein?

von Wolfgang R. (Firma: www.wolfgangrobel.de) (mikemcbike)


Angehängte Dateien:

Lesenswert?

Herrje... Magnetmotoren mit Licht gibt's ja schon... ;-)

von Robert M. (r_mu)


Lesenswert?

Marcel V. schrieb:
> Die Impulskraft ist ziemlich enorm. Eine Rückführung der zweiten Kugel
> an die Startposition, ist dadurch vom Energiegehalt her, also problemlos
> machbar.

Ja natürlich, so wie das 
https://de.wikipedia.org/wiki/Kugelsto%C3%9Fpendel, da ist die 
"Impulskraft" der Kugel die auf die anderen Kugeln draufknallt auch 
ziemlich enorm, darum fliegt die letzte Kugel auch höher als die erste 
und innerhalb kürzester Zeit zerlegt sich der Apparat.

Was glaubt ihr eigentlich warum so eine Kugel nicht in die andere 
eintaucht und unsere Materie "fest" ist? Genau, elektromagnetische 
Kräfte.

von Jens K. (jensky)


Lesenswert?

Einen Vorschlag zur Güte:

Um ihm zu beweisen, dass es nicht möglich ist ein Magnetmotor zu bauen, 
baut ihn nach und demonstriert es einfach! Ansonsten sei ihr nur 
Schwätzer

von Wolfgang R. (Firma: www.wolfgangrobel.de) (mikemcbike)


Lesenswert?

Jens K. schrieb:
> Um ihm zu beweisen, dass es nicht möglich ist ein Magnetmotor zu bauen,
> baut ihn nach und demonstriert es einfach! Ansonsten sei ihr nur
> Schwätzer

Man kann mit einer Konstruktion nicht beweisen, dass es grundsätzlich 
nicht geht.

Man kann mit einer Konstruktion nur beweisen, dass es mit dieser 
Konstruktion nicht geht.

Dein Vorschlag zur Güte ist auch nur Geschwätz.

: Bearbeitet durch User
von Wolfgang R. (Firma: www.wolfgangrobel.de) (mikemcbike)


Lesenswert?

Marcel V. schrieb:
> Die Impulskraft ist ziemlich enorm.

Schon drüber nachgedacht, woher der Impuls entsteht? Welche mechanische 
Energie benötige ich wohl, um die erste magnetisch angezogene Kugel im 
Magnetfeld zu platzieren?

von Robert M. (r_mu)


Lesenswert?

Jens K. schrieb:
> Um ihm zu beweisen, dass es nicht möglich ist ein Magnetmotor zu bauen,
> baut ihn nach und demonstriert es einfach! Ansonsten sei ihr nur
> Schwätzer

Den Beweis führen muss schon derjenige der die Behauptung aufstellt.

von Steven (muellermilch)


Lesenswert?

Uwe S. schrieb:
> Und zwar wird die max. Geschwindigkeit der Kugel beim drehbaren
> Magneten sogar etwas geringer sein. Weil am Drehpunkt des Magneten
> Reibung entsteht.

Warum genau beeinflusst die Reibung des Drehpunktes die Geschwindigkeit 
der Kugel? Entscheidet der Drehpunkt des Magneten über die 
Geschwindigkeit der Kugel und nicht die Kugel selbst?

Uwe S. schrieb:
> Beim starren Magneten hast du zwar einen kürzeren Weg zum Anziehen der
> Kugel, die Kraft ist aber erheblich größer, und die Richtung stimmt
> auch.

Ich verstehe nicht, waurm die Magnetkraft beim starren Magneten größer 
sein soll als beim ausgerichteten Magneten. Wenn ich diese Positionen in 
FEMM vergleiche, dann steht da beim ausgerichteten Magneten mehr Kraft 
als beim starren Magneten. Das will mir nicht einleuchten.

Uwe S. schrieb:
> weil im Anschlagmoment auch noch die kin. Energie des Magnets
> verloren geht.

Kann ich mir den Anschlagmoment so vorstellen, als wenn an der Kugel 
noch ein Stab hängt und dieser Stab dann beim Rollen gegen die Wand 
knallt?

von Rolf (rolf22)


Lesenswert?

Jens K. schrieb:
> Einen Vorschlag zur Güte:
>
> Um ihm zu beweisen, dass es nicht möglich ist ein Magnetmotor zu bauen,
> baut ihn nach und demonstriert es einfach!

Damit könnte man bestenfalls beweisen, dass man ein bestimmter ähnlicher 
Motor (exakt nachbauen kann man ihn ja logischerweise nicht) nicht 
funktioniert.
Über einen Magnetmotor als solchen sagt so ein Beweis gar nichts aus.

> Schwätzer
Glashaus ...

von Alexander (alecxs)


Lesenswert?

Wolfgang R. schrieb:
> Herrje... Magnetmotoren mit Licht gibt's ja schon... ;-)

Das gibt's nicht. Das ist Magie!

Wenn der Energieerhaltungssatz nicht verletzt werden kann muss die 
Energie irgendwo herkommen. Was Du unter "Nichts" verstehst muss genauer 
beschrieben werden. Es sind für mich unbekannte / unentdeckte Formen von 
Energie denkbbar, die man einfach noch nicht erklären kann.

Wenn Du auf Basis deines Schulwissens die Welt erklären willst, wirst Du 
Dich zwangsläufig in Widersprüchen verlieren.

Lustig ist, frage mal dieselben Leute die Dir was von Wissenschaft und 
Physik vorbeten, woraus das Universum entstanden ist.

Ich bin grundsätzlich auch skeptisch gegenüber YouTube Videos, aber im 
Gegensatz zu Dir schließe ich nichts kategorisch aus. Ein drehender 
Magnetmotor ist nun wirklich nichts neues. Sicher gibt es Probleme mit 
der Reibung im Modell, nichts was sich nicht lösen liese. Aber die 
Magneten sind einfach bereits in ihrer Herstellung zu energieaufwendig 
und wirtschaftlich taugt das Konzept nichts.

von Wolfgang R. (Firma: www.wolfgangrobel.de) (mikemcbike)


Lesenswert?

Alexander schrieb:
> Es sind für mich unbekannte / unentdeckte Formen von
> Energie denkbbar, die man einfach noch nicht erklären kann.

Die Hoffnung stirbt zuletzt...

von Alexander (alecxs)


Lesenswert?


von Wolfgang R. (Firma: www.wolfgangrobel.de) (mikemcbike)


Lesenswert?

Alexander schrieb:
> Bla Bla

Die kosmologische Dimension hilft wohl bei der Bewertung der 
Funktionsunmöglichkeit der hier angefragten Magnetmotoren nicht wirklich 
weiter.

Aber das ist ja der üblicher Scammer-Schwurbel - Nullpunktenergie, 
Dunkle Materie, Neutrinos...

Im Rahmen von Galaxien vielleicht von Relevanz, dein Magnetmotor wird 
hier auf der Erde auf deinem Tisch trotzdem nicht laufen.

von Robert M. (r_mu)


Lesenswert?

Steven schrieb:

> Ich verstehe nicht, waurm die Magnetkraft beim starren Magneten größer
> sein soll als beim ausgerichteten Magneten. Wenn ich diese Positionen in
> FEMM vergleiche, dann steht da beim ausgerichteten Magneten mehr Kraft
> als beim starren Magneten. Das will mir nicht einleuchten.

Wenn ich die Kugel losrollen lasse und sie in der Tat auch losrollt dann 
hat sie potentielle Energie die sich in kinetische Energie umwandelt. 
Diese potentielle Energie stecke ich VORHER schon in die Kugel hinein 
indem ich sie an den Platz bewege an dem ich sie losrollen lasse, dazu 
brauch ich ja Kraft über den Weg den ich die Kugel bewege, muss das 
Magnetfeld überwinden das die Kugel schon anzieht etc...

Man kann den Apparat jetzt sicher so bauen, dass die Kugel einen Teil 
der kinetischen Energie die sie aus der potentiellen erhält beibehält, 
also am Magneten vorbeirollt. Trotzdem ist die potentielle Energie dann 
halt weg, und um die Kugel wieder an den Ausgangspunkt zu bringen muss 
ich diese Arbeit wieder reinstecken.

Das alles funktioniert wie ein Pendel. Wenn ich den Magneten drehbar und 
mit Anschlag baue dann wie ein gekoppeltes Pendel, der Unterschied zum 
Einfachpendel ist dass die Bewegung chaotischer wird und die Gegenkräfte 
in der Aufhängung wirklich unübersichtlich werden. Siehe 
https://de.wikipedia.org/wiki/Doppelpendel, der Artikel zeigt auch dass 
das mathematisch recht unübersichtlich wird und man die Bewegungen nicht 
mehr so einfach intuitiv mitverfolgen bzw. vorhersagen kann.

von Robert M. (r_mu)


Lesenswert?

Die hier https://stornetic.com/ entwickeln was was den "Magnetmotoren" 
vermutlich am nächsten kommt (magnetische Lagerung einer Schwungmasse im 
Vakuum)

von Marcel V. (mavin)


Angehängte Dateien:

Lesenswert?

Robert M. schrieb:
> Ja natürlich, so wie das
> https://de.wikipedia.org/wiki/Kugelsto%C3%9Fpendel, da ist die
> "Impulskraft" der Kugel die auf die anderen Kugeln draufknallt auch
> ziemlich enorm, darum fliegt die letzte Kugel auch höher als die erste
> und innerhalb kürzester Zeit zerlegt sich der Apparat.

Die letzte Kugel kann nicht höher fliegen als die erste Kugel, weil 
keine magnetische Unterstützung vorhanden ist. Ganz im Gegensatz zu 
diesem Video:

https://youtu.be/fdoLXChlJ_4?si=fdrdLwQJxckshClu

Im Video wird mit der übertragenen Impulskraft auf die zweite Kugel 
sogar ein Styroporklotz zerschmettert!

Jetzt braucht man nur noch sein Gehirn einschalten, dann wird sofort 
jedem Deppen klar, dass man die abgefeuerte Kugel, statt auf den 
Styroporklotz zu schießen, nur noch über ein Schienensystem zur 
Startposition rollen lassen muss. Dort wird sie automatisch von den 
Permanentmagneten beschleunigt und stößt die vorherige Kugel an, die 
bereits am Ende der Beschleunigungsbahn an der Abschussposition steht.

Das ganze läuft dann ewig im Kreis und es bleibt sogar noch eine 
gehörige Portion Energie zum Abzapfen übrig!

von Wolfgang R. (Firma: www.wolfgangrobel.de) (mikemcbike)


Lesenswert?

Marcel V. schrieb:
> gehörige Portion Energie zum Abzapfen

Wo kommt die her?

von Robert M. (r_mu)


Lesenswert?

Marcel V. schrieb:
> Die letzte Kugel kann nicht höher fliegen als die erste Kugel, weil
> keine magnetische Unterstützung vorhanden ist. Ganz im Gegensatz zu
> diesem Video:
>
> https://youtu.be/fdoLXChlJ_4?si=fdrdLwQJxckshClu
>
> Im Video wird mit der übertragenen Impulskraft auf die zweite Kugel
> sogar ein Styroporklotz zerschmettert!

Ja wirklich wild. Man sieht auch recht deutlich dass die Stahlkugel dann 
am Magnet festklebt, um die dort wieder wegzubringen muss ich mehr 
Arbeit reinstecken als die Glasmurmel vorher ins Styropor geknallt hat.

von Wolfgang R. (Firma: www.wolfgangrobel.de) (mikemcbike)


Lesenswert?

Marcel V. schrieb:
> dann wird sofort
> jedem Deppen klar

Dass das nicht funktioniert. Die abgeschossene Kugel ist aus gutem Grund 
nicht magnetisch. Die Energie für das Abschießen der Styroporklötze wird 
aus der potentiellen Energie der Stahlkugel am Bahnanfang entnommen.

von Robert M. (r_mu)


Lesenswert?

Marcel V. schrieb:
> Jetzt braucht man nur noch sein Gehirn einschalten, dann wird sofort
> jedem Deppen klar, dass man die abgefeuerte Kugel, statt auf den
> Styroporklotz zu schießen, nur noch über ein Schienensystem zur
> Startposition rollen lassen muss. Dort wird sie automatisch von den
> Permanentmagneten beschleunigt und stößt die vorherige Kugel an, die
> bereits am Ende der Beschleunigungsbahn an der Abschussposition steht.

Versuch macht kluch. Dummerweise wird das immer an irgendwelchen 
Kleinigkeiten scheitern, man steht dann quasi immer vorm Durchbruch, es 
funktioniert "fast" etc... so Beispiele von Leuten die das erst nach 
einem Haufen Geld und Zeit eingesehen haben gibts auf youtube zu Hauf, 
hier z.B. https://www.youtube.com/@dietmarhohl

(wobei da vermutlich keine Einsicht vorhanden ist sondern man einfach 
nur aus Frust aufgegeben hat)

von Uwe S. (bullshit-bingo)


Lesenswert?

Steven schrieb:
> Ich verstehe nicht, waurm die Magnetkraft beim starren Magneten größer
> sein soll als beim ausgerichteten Magneten.

Nicht die Magnetkraft. Die Kraft, die tatsächlich zur Drehung des Rotors 
führt, oder zur Bewegung der Kugel. Die ist maßgeblich von der Richtung 
der Krafteinwirkung abhängig.

Steven schrieb:
> Warum genau beeinflusst die Reibung des Drehpunktes die Geschwindigkeit
> der Kugel?

Es entsteht doch zusätzliche Reibung, die ja am Ende nur der Kugel 
fehlen kann.
Genau genommen bremst der reibungsbehaftete Magnet die Kugel schon 
während der Beschleunigung leicht aus. Bzw. sorgt dadurch einfach für 
weniger Maximalgeschwindigkeit. Stell` dir den Magnet in Honig gelagert 
vor, dann siehst du auch gedanklich eine Kugel, die nur ganz langsam 
heranrollt...

Steven schrieb:
>> weil im Anschlagmoment auch noch die kin. Energie des Magnets
>> verloren geht.
>
> Kann ich mir den Anschlagmoment so vorstellen, als wenn an der Kugel
> noch ein Stab hängt und dieser Stab dann beim Rollen gegen die Wand
> knallt?

Nein, betrachte einfach den Magneten, der gegen die Wand knallt. Da geht 
doch Energie verloren. Auch diese muss der Kugel fehlen.

von Marcel V. (mavin)


Lesenswert?

Wolfgang R. schrieb:
> Wo kommt die her?

Ganz einfach, die Energie könnte aus der enorm hohen kinetischen Energie 
aus der Kugel kommen, oder aus der potentiellen Energie, die ebenfalls 
in der Kugel gespeichert ist, wenn sie aus einer höheren Position heraus 
zur Startposition rollt. Dafür müsste man die Kugelbahn nur etwas schräg 
stellen, so dass die Kugel schräg nach oben abgefeuert wird.

Das dürfte also nicht allzu schwer sein und sollte an einem Nachmittag 
fluxartig aufgebaut sein!

Fertig ist das Perpetuum Mobile.

von Alexander (alecxs)


Lesenswert?

Finde den Fehler
https://youtu.be/z2K9MbvYsmk

von Robert M. (r_mu)


Lesenswert?

Marcel V. schrieb:
> Ganz einfach, die Energie könnte aus der enorm hohen kinetischen Energie
> aus der Kugel kommen, oder aus der potentiellen Energie, die ebenfalls
> in der Kugel gespeichert ist, wenn sie aus einer höheren Position heraus
> zur Startposition rollt. Dafür müsste man die Kugelbahn nur etwas schräg
> stellen, so dass die Kugel schräg nach oben abgefeuert wird.

Die Kugel hat auch potentielle Energie im Magnetfeld, die steckt man 
hinein indem man die Kugel dorthin positioniert wo man sie auslässt. 
Diese potentielle Energie kann ich in Bewegungsenergie (kinetische 
Energie) umwandeln. Hab ich die Kugel beschleunigt ist entsprechend 
potentielle Energie weg. Im Prinzip (bis auf Reibung und andere 
Verluste) ist das auch reversibel, kann also sehr lange hin und her 
pendeln wenn geschickt gebaut. Mehr rauskommen als man reinsteckt tut 
aber nicht.

von Steven (muellermilch)


Lesenswert?

Uwe S. schrieb:
> Nicht die Magnetkraft. Die Kraft, die tatsächlich zur Drehung des Rotors
> führt, oder zur Bewegung der Kugel. Die ist maßgeblich von der Richtung
> der Krafteinwirkung abhängig.

Aber ist nicht die Anziehungskraft genau die Kraft die zur Bewegung der 
Kugel führt? Welche Kraft bewegt denn bei dir die Kugel? Ich verstehe 
dich einfach nicht.

Uwe S. schrieb:
> Stell` dir den Magnet in Honig gelagert
> vor, dann siehst du auch gedanklich eine Kugel, die nur ganz langsam
> heranrollt...

Ich soll mir den dynamischen Magneten in Honig gelagert vorstellen, 
verstehe ich das richtig? Der dynamische Magnet dreht sich also sehr 
langsam. Das ist doch aber der Kugel egal. Die Kugel passt sich doch 
nicht der Drehung des Magneten an, sondern der Anziehungskraft. Deswegen 
rollt die Kugel doch nicht langsamer. Das erscheint mir unlogisch oder 
ich verstehe deine Aussage nicht.

: Bearbeitet durch User
von Robert M. (r_mu)


Lesenswert?

Marcel V. schrieb:
> Die letzte Kugel kann nicht höher fliegen als die erste Kugel, weil
> keine magnetische Unterstützung vorhanden ist.

Das Kugelstosspendel kann man beliebig mit Magneten modifizieren um da 
eine "magnetische Unterstützung" einzubauen, das Verhalten wird trotzdem 
nicht viel anders sein.... einmal angestoßen pendeln die Kugeln eine 
Zeit lang hin und her, die Auslenkungen nehmen ab, und irgendwann steht 
wieder alles.

von Peter N. (alv)


Lesenswert?

Florian schrieb:
> Wenn jemand behauptet, er könnte aus 52 Spielkarten nur durch Mischen
> und ohne Tricks 53 Spielkarten machen, muss man nicht erst den Fehler
> suchen.

Naja, wenn man den Abrieb der Karten beim Mischen sammelt und daraus 
dann eine neue Karte zusammenbackt...

von Peter N. (alv)


Lesenswert?

Was mich bei dieser ganzen Diskussion intressiert: was ist Magnetismus 
überhaupt?

Die Modellvorstellung mit den Molekularmagneten ist ganz schön, führt 
aber nicht weiter.
Wo aus dem magnetischen Molekül/Atom kommt die magnetische Feldlinie?
Wieso geschieht das nur bei wenigen Materialien?

von Robert M. (r_mu)


Lesenswert?

Peter N. schrieb:
> Was mich bei dieser ganzen Diskussion intressiert: was ist Magnetismus
> überhaupt?
>
> Die Modellvorstellung mit den Molekularmagneten ist ganz schön, führt
> aber nicht weiter.
> Wo aus dem magnetischen Molekül/Atom kommt die magnetische Feldlinie?
> Wieso geschieht das nur bei wenigen Materialien?

Magnetismus hat immer was mit bewegten Ladungen zu tun. Im Extremfall 
dem Eigendrehimpuls von (geladenen) Elementarteilchen. Das Magnetfeld 
ist aber nichts anderes als ein anders betrachtetes elektrisches Feld -- 
die Dinger gehen fliessend ineinander über, je nachdem wo und wie man 
das anschaut.

Wo die Feldlininen anfangen / enden wird irgendwann immer unschärfer je 
genauer man hinschaut. Heisenberg und so.

Magnetismus im Festkörper ist eine recht komplizierte Geschichte, ich 
find die Modellvorstellung mit den Elementarmagneten für klassische 
Betrachtungen nicht so verkehrt.

von Dieter D. (Firma: Hobbytheoretiker) (dieter_1234)


Lesenswert?

Wolfgang R. schrieb:
> Marcel V. schrieb:
>> gehörige Portion Energie zum Abzapfen
>
> Wo kommt die her?

Sehr viele Bohnen essen. ;)

So faehrt ein Auto ohne Sprit:
https://g.co/about/xy7dda
Genial!

von Monk (roehrmond)


Lesenswert?

Jens K. schrieb:
> Um ihm zu beweisen, dass es nicht möglich ist ein Magnetmotor zu bauen,
> baut ihn nach und demonstriert es einfach! Ansonsten sei ihr nur
> Schwätzer

Ich erwarte mit Freude die Präsentation deines Magnetmotors. Denn du 
bist ja ein Macher, kein Schwätzer.

: Bearbeitet durch User
von Uwe S. (bullshit-bingo)


Lesenswert?

Steven schrieb:
> Aber ist nicht die Anziehungskraft genau die Kraft die zur Bewegung der
> Kugel führt?

Ja, aber die Richtung der Anziehungskraft muss auch stimmen. Die Kugel 
rollt ja nicht direkt zum Magneten hin, sondern soll über ihn hinweg 
rollen. Da ist die resultierende Kraft deutlich kleiner. Besonders zum 
Ende des Anziehungsvorgangs hin, obwohl dort die Anziehungskraft am 
größten ist.

Steven schrieb:
> Die Kugel passt sich doch
> nicht der Drehung des Magneten an, sondern der Anziehungskraft.

Oh doch, sie passt sich schon teilweise der Drehung des Magneten an. Das 
mit dem in Honig schwer drehbaren Magnet ist ja nur ein Extrembeispiel, 
um dir die Vorgänge zur Verlangsamung der Kugel zu demonstrieren. Wird 
der Magnet gebremst, kommt die Kugel garantiert etwas später im Zentrum 
der Anordnung an. Warum es die Kugel genau verlangsamt, ist aber 
eigentlich egal. Es wird am Magnet Reibung erzeugt und seine kin. 
Energie wird am Anschlag verheizt. Das bedeutet auf jeden Fall weniger 
Ausschlag der Kugel über die Mitte hinaus. Denn sie ist ja abseits des 
Magneten das einzige sich bewegende Teil.

von Joe G. (feinmechaniker) Benutzerseite


Lesenswert?

Eine feste Überzeugung reicht leider nicht in der Wissenschaft ;-)
Versuche es mal mit einer Beweisumkehr. Du gehst das von aus, dass kein 
PM existiert und der Energieerhaltungssatz in abgeschlossenen Systemen, 
sowie der 2. HS. gültig sind. Jetzt stellst du alle zugehörigen 
Bilanzgleichungen der Bewegung auf, und suchst nach der Verletzung der 
obigen Gesetze. Wenn du alles richtigmachst, hast du bei dieser 
Herangehensweise etwas gelernt.

von Peter N. (alv)


Lesenswert?

Robert M. schrieb:
> Magnetismus hat immer was mit bewegten Ladungen zu tun. Im Extremfall
> dem Eigendrehimpuls von (geladenen) Elementarteilchen.

Ok, irgendwo bewegt sich ein Elektron in einer Kreisbahn. Das entspräche 
einem Stromfluß in einer Spule, was ein Magnetfeld erzeugt.

Warum funktioniert das aber nur bei Eisen (und wenigen anderen 
Materialien)?

Sind Eisenatome grundsätzlich Elementarmagnete, im Normalfall aber so 
durcheinander angeordnet, daß nach Außen hin kein Magnetfeld wirksam 
ist?

von Steven (muellermilch)


Lesenswert?

Uwe S. schrieb:
> Ja, aber die Richtung der Anziehungskraft muss auch stimmen.

Und die Richtung der Anziehungskraft ist am besten beim dynamischen 
Magneten.

Uwe S. schrieb:
> Die Kugel rollt ja nicht direkt zum Magneten hin, sondern soll über ihn
> hinweg rollen

Die Kugel rollt schon direkt zum Magneten hin, und zwar genau da hin, wo 
die stärkste Anziehungskraft wirkt. Die Kugel rollt nur weiter, weil sie 
in dem Moment einen Energieüberschuss hat. Die Energie muss ja irgendwo 
hin. Diese wird beim Bremsvorgang wieder verbraucht bzw. umgewandelt.

Ich verstehe deine Erklärung oder Logik einfach nicht, warum die 
Ausrichtung des Magneten keinen Einfluß auf die Geschwindigkeit der 
Kugel haben soll im Vergleich zum statischen Magneten. Der Magnet 
richtet sich aus, weil er die stärkste Anziehungskraft sucht. Auf die 
Kugel wirkt also eine höhere Anziehungskraft als beim statischen 
Magneten.
Das die Drehung des Magneten und der stoß gegen die Wand einen Einfluss 
hat, sehe ich ein, aber bei der Ausrichtung fehlt mir die richtige 
Erklärung, warum die Ausrichtung der Anziehungskraft keinen Vorteil 
bringt.

Ich plane beide Fälle nachzubauen und schaue es mir praktisch an. Mit 
entsprechenden Videos versuche ich dann die Geschwindigkeit der Kugel 
für beide Fälle zu bestimmen und zu vergleichen.

von Giovanni (sqrt_minus_eins)


Lesenswert?

Meta-Diskussion:

Wie hoch ist eigentlich der Energieaufwand (=Verschwendung?) durch 
diesen Beitrag?
* Verbrauch der Server + Netzinfrastruktur
* Verbrauch PC/...  der Nutzer - die nur Lesen oder auch Schreiben
* Kann die vorgeschlagene "Idee" diesen Aufwand jemals kompensieren?
* Oder verbraucht die "Idee" selber Energie? Vielleicht ist in der Idee 
ein Vorzeichenfehler, und man muss Energie reinstecken, damit sich das 
Ding dreht.

Ev. auch der volkswirtschaftliche Schaden durch Nutzer, die wären der 
Arbeitszeit *) Kommentare schreiben (es gilt die Unschuldsvermutung).

*) Annahme: es nehmen nicht nur Rentner und Kinder an der Diskussion 
teil

von Robert M. (r_mu)


Lesenswert?

Peter N. schrieb:
> Ok, irgendwo bewegt sich ein Elektron in einer Kreisbahn. Das entspräche
> einem Stromfluß in einer Spule, was ein Magnetfeld erzeugt.

Mit Eigendrehimpuls meint man eher dass sich das Teilchen um die eigene 
Achse dreht. Was so direkt natürlich nicht der Fall ist.

> Warum funktioniert das aber nur bei Eisen (und wenigen anderen
> Materialien)?
>
> Sind Eisenatome grundsätzlich Elementarmagnete, im Normalfall aber so
> durcheinander angeordnet, daß nach Außen hin kein Magnetfeld wirksam
> ist?

Alle geladenen Elementarteilchen sind Elementarmagnete. Das magnetische 
Moment eines Atoms ist die Summe der magnetischen Momente der Elektronen 
und ihrer Bahnen und des Moments des Kerns. Das sagt für sich genommen 
so aber noch nicht ob das dann ferro para oder diamagnetisch ist.

Für Ferromagnetismus brauchts mehrere Atome AFAIK, das ist ein 
"ensemble"-Phänomen. Wie gesagt das ist alles recht kompliziert und ich 
maße mir nicht an das jemandem erklären zu können sondern verweise auf 
wikipedia: 
https://de.wikipedia.org/wiki/Austauschwechselwirkung#Austauschenergie_und_magnetische_Ordnung

: Bearbeitet durch User
von John B. (craftsman)


Lesenswert?

Steven schrieb:
> Ich plane ...

"morgen, ja morgen, fängt ein neues Leben an ..." :)

Wenn du so experimentierst, wie du hier diskutierst, wirst du die 
Ergebnisse ebensowenig zur Kenntnis nehmen, wie den Inhalt der Antworten 
hier. Immer wieder kann man Gründe finden, warum es jetzt grad noch 
nicht geklappt hat, und man kann sich immer wieder stur stellen und 
meinen, "aber beim nächsten Mal wird es ganz sicher klappen".

Ich fürchte, du bist kein Troll, der in harmloser Weise Spass daran hat, 
alle Erklärungsversuche ins leere laufen zu lassen. Ich fürchte, du bist 
süchtig danach, dir durch Ausblenden von Teilen der Realität die 
Illusion zu verschaffen, gescheiter zu sein, als die Natur und ihre 
Beschreibung durch die Physik.

von Carypt C. (carypt)


Lesenswert?

Was kostet die Gruppentherapie, ja, da fragt nicht nach dem Preis, wenns 
denn hilft. Manchen ist ja schon geholfen wenn sie eine feste 
Tagesstruktur haben, eine Beschäftigung, ein Hobby. Es ist ja auch eine 
Kulturleistung. Man lernt und lehrt, man hilft Einander, wie schön.

Natürlich hat die beste Ausrichtung des Magneten einen Vorteil. 
Allerdings nimmt die Magnetkraft mit der Entfernung ziemlich ab.

von Dieter D. (Firma: Hobbytheoretiker) (dieter_1234)


Lesenswert?

Giovanni schrieb:
> Ev. auch der volkswirtschaftliche Schaden durch Nutzer,

Sicherlich geringer als das Saufen, Rauchen oder Kiffen anderer, oder 
sich nochn Riegel reinschieben und vor Gewicht kaum noch gehen koennen. 
Dann schon besser hier unterwegs sein.

von Dieter D. (Firma: Hobbytheoretiker) (dieter_1234)


Lesenswert?

Vielleicht klappt irgendwann ein Aufbau des TO und wird ein Nachfolger 
von Deadalus. Dem Erbauer von Perpetums, denen das Harvesting nicht mehr 
anzusehen ist.

Statt Licht kann auch IR-Strahlung als Antrieb genutzt werden. Dito auch 
fuer gering unterschiedliche Waermepotentiale. Mit Magneten und 
Halleffekt kann der Strom auch kommutiert werden.

von H.Joachim S. (crazyhorse)


Lesenswert?

Dieter D. schrieb:
> Saufen, Rauchen oder Kiffen anderer, oder
> sich nochn Riegel reinschieben und vor Gewicht kaum noch gehen koennen

Hehe, das hat Potential für allgemeine Entrüstung und Empörung. Du hast 
jetzt einfach mal so mehr als die Hälfte Deutschlands diskriminiert oder 
beleidigt :-)


Ansonsten: das könnte hier mal langsam geschlossen werden. Nicht 
deswegen, sondern weil das Thema so unergiebig ist.

von Alexander (alecxs)


Lesenswert?

Robert M. schrieb:
> Die hier https://stornetic.com/ entwickeln was was den "Magnetmotoren"
> vermutlich am nächsten kommt (magnetische Lagerung einer Schwungmasse im
> Vakuum)

Es wird auf jeden Fall ein Teil des Tricks werden.

Alexander schrieb:
> Finde den Fehler
> https://youtu.be/z2K9MbvYsmk

Keiner?

von Re D. (re_d228)


Lesenswert?

Dieter D. schrieb:
> Sicherlich geringer als das Saufen, Rauchen oder Kiffen anderer, oder
> sich nochn Riegel reinschieben und vor Gewicht kaum noch gehen koennen.
> Dann schon besser hier unterwegs sein.

Wie sieht es mit Typen aus, die Fremde Leute in der S-Bahn 
vollquatschen?

von ●DesIntegrator ●. (Firma: FULL PALATINSK) (desinfector) Benutzerseite


Lesenswert?

Freie Energie.
Man halte einen Magnet an eine Leuchtstoffröhre im Betrieb.
An der Stelle des Magnet wird das licht heller.

.
.
. . .  WOW!!!
.
.

von Florian (flori_n)


Lesenswert?

Steven schrieb:
> Das die Drehung des Magneten und der stoß gegen die Wand einen Einfluss
> hat, sehe ich ein, aber bei der Ausrichtung fehlt mir die richtige
> Erklärung, warum die Ausrichtung der Anziehungskraft keinen Vorteil
> bringt.
Was heißt 'Vorteil'? In verschiedenen Phasen des Ablaufs werden sich die 
Geschwindigkeiten der Kugel zwischen den beiden Konfigurationen schon 
ein bisschen unterscheiden. Wie beim Beispiel der Murmelbahn kann man 
die Bahn auch flacher oder steiler machen und so die Geschwindigkeit der 
Kugel ändern.
Allerdings war die Idee ja, wenn ich das richtig verstanden habe, mit 
einer besonderen Anordnung Überschussenergie zu bekommen, und das ist 
weder beim Magneten, noch bei der Kugelbahn möglich.

Das Problem bei den Gedanken zum Magneten ist, dass sich das Magnetfeld 
ständig ändert, wenn sich Magnet oder Kugel bewegen. Deshalb reicht es 
nicht das Magnetfeld und die resultierenden Kräfte zu wenigen 
Zeitpunkten zu betrachten, sondern man müsste das für den gesamten 
Vorgang berechnen und die Kraft integrieren. Das ist halt nicht mehr 
intuitiv und selbst mit Computern relativ aufwendig.

Für die weitere Diskussion: Hast du verstanden oder zumindest 
akzeptiert, dass du aus der (oder einer anderen) Anordnung keinen 
Energieüberschuss rausnehmen kannst?

PS: 'Die Energie wird aus den Magneten entnommen, die dadurch mit der 
Zeit schwächer werden'. Mal abwarten, bis jemand auf die Idee kommt, 
eine Perpetuum-Murmelbahn zu bauen, die die Energie aus dem 
Gravitationsfeld der Erde entnimmt, das dadurch im Laufe der Zeit ein 
bisschen schwächer wird.

von John B. (craftsman)


Lesenswert?

Carypt C. schrieb:
> Gruppentherapie

Und?

Wer hat denn Bedarf an Therapie, deiner Meinung nach?

von Uwe S. (bullshit-bingo)


Lesenswert?

Florian schrieb:
> In verschiedenen Phasen des Ablaufs werden sich die
> Geschwindigkeiten der Kugel zwischen den beiden Konfigurationen schon
> ein bisschen unterscheiden.

Ja, deutlich sogar. Beim bewegten Magneten wird die Beschleunigung 
früher erfolgen, dafür aber während des gesamten Vorgangs schwächer. 
Weil eben die Richtung der Anziehungskraft fast nie mit der Richtung der 
Kugel übereinstimmt. Beim starren Magneten wird die Kugel erst später 
angezogen, aber dafür mit hoher Kraft. Also wäre die Kugel direkt überm 
Magneten gleich schnell, wenn da nicht die Reibung und die verlorene 
kin. Energie des Magneten wären. Die können nur der Kugel fehlen, da ja 
ansonsten keine Energie verloren gehen darf.

Steven hat sich leider so sehr auf die drehbaren Magneten eingeschossen, 
daß auch die 17. Erklärung nicht ausreichen würde. Immerhin scheint er 
sich vom Gedanken getrennt zu haben, die Energieprobleme der Welt 
innerhalb 3 Minuten gelöst zu haben.

von Robert M. (r_mu)


Lesenswert?

Uwe S. schrieb:
> Ja, deutlich sogar. Beim bewegten Magneten wird die Beschleunigung
> früher erfolgen, dafür aber während des gesamten Vorgangs schwächer.
> Weil eben die Richtung der Anziehungskraft fast nie mit der Richtung der
> Kugel übereinstimmt. Beim starren Magneten wird die Kugel erst später
> angezogen, aber dafür mit hoher Kraft. Also wäre die Kugel direkt überm
> Magneten gleich schnell, wenn da nicht die Reibung und die verlorene
> kin. Energie des Magneten wären. Die können nur der Kugel fehlen, da ja
> ansonsten keine Energie verloren gehen darf.
>

Es ist auch die potentielle Energie der Kugel beim Start in den beiden 
Situationen nicht die selbe, schon alleine daher wird sich das 
unterschiedlich verhalten.

von Steven (muellermilch)


Lesenswert?

Uwe S. schrieb:
> Beim bewegten Magneten wird die Beschleunigung früher erfolgen, dafür
> aber während des gesamten Vorgangs schwächer. Weil eben die Richtung der
> Anziehungskraft fast nie mit der Richtung der Kugel übereinstimmt.

Und genau das verstehe ich nicht. Diese Erklärung leuchtet mir nicht.
Natürlich stimmt die Richtung der Anziehungskraft mit der Kugel überein. 
Der bewegte Magnet passt sich der Kugel doch immer an. Warum sollte die 
Richtung der Anziehungskraft des Magneten nicht mit der Kugel 
übereinstimmen?
Der starre Magnet zeigt mit dem Zeigefinger nach oben, der bewegte 
Magnet folgt stets mit dem Zeigefinger die Kugel. Deswegen möchte ich es 
bauen und messen, entweder um zu beweisen, dass diese Aussage falsch ist 
oder sie zu akzeptieren, dass sie richtig ist.
Es sei denn, jemand ist in der Lage mir anders zu erklären, warum die 
Kugel mit dem starren Magneten schneller ist als mit dem bewegten 
Magneten. Uwe seine Erklärung verstehe ich leider nicht.

Florian schrieb:
> Hast du verstanden oder zumindest akzeptiert, dass du aus der (oder
> einer anderen) Anordnung keinen Energieüberschuss rausnehmen kannst?

Das akzeptiere ich erst, wenn ich den Unterschied zwischen dem starren 
und dem bewegten Magneten verstehe oder sehe. Kennt denn wer von euch 
Beispiele wo soetwas schon verglichen oder getestet wurde? Ich kann mich 
mit der Praxis besser anfreunden als mit der Theorie. Mit der Theorie 
habe ich es nicht so, wie ihr merkt ;)

von Wolfgang R. (Firma: www.wolfgangrobel.de) (mikemcbike)


Lesenswert?

Immerhin hat mich dieser Thread mal dazu gebracht, mir die 
Kommentarspalten  bei diversen Youtube Filmen anzusehen - da verliert 
man echt den Glauben an die Menschheit. Eine große Anzahl an Menschen 
lebt wohl nach der Maxime "Belästige mich nicht mit Fakten!" Da ist jede 
physiklische Erklärung sinnlos, weil man ja will, dass es funktioniert. 
Psyche statt Physik.

von Alexander (alecxs)


Lesenswert?

Und Du gehörst dazu. Denn du gibst dich mit ner vorgekauten Erklärung 
zufrieden, der Fehler in der Betrachtung ist dir nicht aufgefallen.

von Wolfgang R. (Firma: www.wolfgangrobel.de) (mikemcbike)


Lesenswert?

Alexander schrieb:
> der Fehler in der Betrachtung ist dir nicht aufgefallen.

Na dann kläre uns mal bitte auf, welcher Fehler in der Betrachtung dazu 
führt, dass Magnetmotoren plötzlich doch funktionieren.

Selbst der Kanalbetreiber sagt eindeutig, dass nicht alle genannten 
Analogien 100% korrekt sind.

von Bruno V. (bruno_v)


Lesenswert?

Carypt C. schrieb:
> Klar könnte man sagen, Druckluft, aber falls nicht

.. dann war es was anderes, wie in der 4.ten Antwort

>> H.Joachim S. schrieb:
>> Die schöpferische Leistung besteht immer nur darin
>> den Energieeintrag mehr oder weniger gut zu verschleiern.

> kein Perpetuum Mobile sei, weil Magnete in 100 Jahren 5% schwächer werden

1 Sekunde reicht, wenn er Reibverluste überwindet ohne dass er 
kinetische, potentielle, chemische, elektrische oder andere "klassische" 
Energie benötigt bzw. verliert. Dann bekommst Du 1 Millionen aus 
Stockholm und vorab innerhalb von 24 Stunden alle mathematischen und 
physikalischen Erklärungen kostenfrei.

(Aus praktischen Gründen sollten es ein paar Minuten sein, sonst kommt 
keiner raus)

: Bearbeitet durch User
von Uwe S. (bullshit-bingo)


Lesenswert?

Steven schrieb:
> Natürlich stimmt die Richtung der Anziehungskraft mit der Kugel überein.

Nein, der Magnet will die Kugel nach unten ziehen, die Oberfläche zwingt 
sie aber zu einer waagerechten Bahn.
Das will der starre Magnet zwar auch, aber er setzt wenigstens zu Anfang 
nahezu vor der Kugel an, nicht nur fast unter ihr. Der bewegte Magnet 
ist zunächst einfach sehr weit von der Kugel entfernt, aber sobald er 
wirklich Kraft ausüben kann, rollt sie quasi nur noch über ihm. Das ist 
beim starren Magneten anders.
Wie gesagt, beides Mal wird die Kugel dieselbe Maximalgeschwindigkeit 
haben, aber eben abzüglich der Energieverluste durch den bewegten 
Magneten.
Du brauchst wirklich rein gar nichts zu testen, es ist sehr 
offensichtlich.
Du musst eigentlich nur verstehen, daß man, wollte man ein Auto 
anschieben, dies nicht durch optimierte Handauflagen am Seitenfenster 
erreicht, sondern durch einfachen Schub vom Heck aus. Ich muss es so 
albern sagen, denn du erkennst nicht, wie eine Kraft gerichtet sein 
muss, die die Kugel tatsächlich beschleunigt. Diese Kraft wirkt nicht 
unter der Kugel, sondern würde von vorn an ihr ziehen. Ist das wirklich 
so schwer?

von Robert M. (r_mu)


Lesenswert?

Steven schrieb:
> Florian schrieb:
>> Hast du verstanden oder zumindest akzeptiert, dass du aus der (oder
>> einer anderen) Anordnung keinen Energieüberschuss rausnehmen kannst?
>
> Das akzeptiere ich erst, wenn ich den Unterschied zwischen dem starren
> und dem bewegten Magneten verstehe oder sehe. Kennt denn wer von euch
> Beispiele wo soetwas schon verglichen oder getestet wurde? Ich kann mich
> mit der Praxis besser anfreunden als mit der Theorie. Mit der Theorie
> habe ich es nicht so, wie ihr merkt ;)

Es ist vollkommen egal wie die Kugel da genau beschleunigt wird. Man 
kann da das Experiment ungewollt in verschiedene Richtungen 
beeinflussen, z.B. machts einen Unterschied ob die Kugel rollt oder 
rutscht, und das wiederum kann von der Anziehungskraft des Magneten 
abhängen.

Die Krux an der Sache ist, dass die Kugel nicht von selbst in die 
Ausgangsposition kommt, die muss man dorthin bewegen, und dabei 
verrichtet man Arbeit gegen das Magnetfeld die dann in der Kugel steckt, 
das nennt man potentielle Energie. Wenn ich die Kugel auslasse 
entspricht das dem Loslassen eines ausgelenkten Pendels. Die Kugel kann 
dann je nach Vorrichtung einen Teil dieser potentiellen Energie in 
Bewegungsenergie umwandeln, und einen Teil dieser Bewegungsenergie auch 
wieder in potentielle Energie, aber sie kann aus freien Stücken nicht 
wieder zum Ausgangspunkt zurück.

Aber bau den Versuch auf, ist sicher lehrreich.

: Bearbeitet durch User
von Georg M. (g_m)


Angehängte Dateien:

Lesenswert?

Man muss nur glauben.

von Alexander (alecxs)


Lesenswert?

Wolfgang R. schrieb:
> Na dann kläre uns mal bitte auf, welcher Fehler in der Betrachtung dazu
> führt, dass Magnetmotoren plötzlich doch funktionieren.

Einfach mal die Rolle abwickeln in eine Ebene? Stator- und 
Rotormassenverhältnis flux vertauschen? Massenträgheit komplett 
vernachlässigen? Darauf fallen nur Psiramjünger rein.

von Robert M. (r_mu)


Lesenswert?

Alexander schrieb:
> Wolfgang R. schrieb:
>> Na dann kläre uns mal bitte auf, welcher Fehler in der Betrachtung dazu
>> führt, dass Magnetmotoren plötzlich doch funktionieren.
>
> Einfach mal die Rolle abwickeln in eine Ebene? Stator- und
> Rotormassenverhältnis flux vertauschen? Massenträgheit komplett
> vernachlässigen? Darauf fallen nur Psiramjünger rein.

Was bestimmt bei so einem "funktionierendem" Magnetmotor eigentlich die 
Rotationsgeschwindigkeit?

von Wolfgang R. (Firma: www.wolfgangrobel.de) (mikemcbike)


Lesenswert?

Alexander schrieb:
> Darauf fallen nur Psiramjünger rein.

Klar. Dann bin ich ja gespannt, wann mir einer der erleuchteten 
Psiram-Leugner den ersten funktionsfähigen Magnetmotor präsentiert.

Fast geschaft...

Muss nur noch ein wenig optimiert werden, dann läuft's.

Ist zu klein, funktioniert nur größer.

Zu langsam...

Zu schnell...

Zu leicht...

Mir fehlt Geld, um den Durchbruch zu erreichen.

Gibt mir jemand Geld?

von Steven (muellermilch)


Angehängte Dateien:

Lesenswert?

Uwe S. schrieb:
> Nein, der Magnet will die Kugel nach unten ziehen, die Oberfläche zwingt
> sie aber zu einer waagerechten Bahn.
> Das will der starre Magnet zwar auch, aber er setzt wenigstens zu Anfang
> nahezu vor der Kugel an, nicht nur fast unter ihr. Der bewegte Magnet
> ist zunächst einfach sehr weit von der Kugel entfernt, aber sobald er
> wirklich Kraft ausüben kann, rollt sie quasi nur noch über ihm. Das ist
> beim starren Magneten anders.

Das Magnetfeld des bewegten Magneten sieht grob dargestellt so aus: 
siehe Bild.

Zu betrachten ist, dass die Magnetkraft an der Kugel hier am Größten ist 
im Vergleich zum starren Magneten. Ich hoffe da sind wir uns einig. 
Ansonsten packe ich FEMM aus und zeige dir die Kraftwerte.
Deine Annahme ist, aufgrund des Winkels des bewegten Magneten, dass die 
nach unten ziehende Kraft größer ist als beim starren Magneten. Das 
wiederum bedeutet, der Reibungsverlust ist hier auch größer. Diese 
Annahme gehe ich mit. Allerdings behaupte ich, dass hier trotzdem die 
ziehende Wirkung größer ist als beim starren Magneten, weil die 
Magnetkraft auf die ziehende Seite der Kugel immernoch größer ist, als 
beim starren Magneten. Auch hier kann ich FEMM auspacken und dir die 
entsprechenden Werte zeigen.
Ich gehe aber mit, dass aufgrund der höheren Reibungsverluste des 
bewegten Magneten, sich die Geschwindigkeiten am Ende ausgleichen. 
Praktisch bewiesen hätte ich es trotzdem gerne. Solange lässt mir das 
keine Ruhe. Da kannst du noch sehr behaupten, dass es offensichtlich 
sei. Du selbst kannst es ja auch nicht beweisen, oder?

von Wolfgang R. (Firma: www.wolfgangrobel.de) (mikemcbike)


Lesenswert?

Alexander schrieb:
> Einfach mal die Rolle abwickeln in eine Ebene?

Ein völlig übliches Vorgehen, um rotierende Systeme anschaulich zu 
erklären.

Alexander schrieb:
> Stator- und
> Rotormassenverhältnis flux vertauschen?

In wieweit sind die Masseverhältnisse von Stator und Rotor essentiell 
wichtig für die Funktion des Perpetuum Mobiles? Gibt es da ein optimales 
Verhältnis? Ist das skalierbar? Inwieweit spielt die masse der Magnete 
eine Rolle?

Alexander schrieb:
> Massenträgheit komplett
> vernachlässigen?

Ist die Massenträgheit verantwortlich für die Funktion? Welche Trägheit 
muss das System besitzen, dass es funktioniert? Wie spielt das dann mit 
zunehmender Rotationsgeschwindigkeit zusammen?

Alexander schrieb:
> Darauf fallen nur Psiramjünger rein.

Aha. Welcher der oben genannten Punkte führt dann letztendlich zum 
Versagen des PM? Vielleicht kann der Kanalbetreiber das ja dann noch 
etwas differenzierter darlegen...

von Wolfgang R. (Firma: www.wolfgangrobel.de) (mikemcbike)


Angehängte Dateien:

Lesenswert?

Steven schrieb:
> Das Magnetfeld des bewegten Magneten sieht grob dargestellt so aus:
> siehe Bild.

Ich hatte Magnetfeldlinien (ungestört) immer anders in Erinnerung...

von Florian (flori_n)


Lesenswert?

Die Diskussion ist so auch schon lang genug, auch ohne das Trollfutter 
für Alexander. Deshalb: Trolle bitte nicht füttern.

Steven schrieb:
> Das Magnetfeld des bewegten Magneten sieht grob dargestellt so aus:
> siehe Bild.
Das könnte der Fehler liegen. So sieht das Magnetfeld nicht aus. 
Wolfgang hat den richtigen Verlauf gezeigt.
Recht hättest du, wenn der Magnet ein durch die Zeichnungsebene 
stromdurchflossener Leiter wäre. Das soll aber wohl hier eher nicht der 
Fall sein.

Steven schrieb:
> Ansonsten packe ich FEMM aus
Das ist eine gute Idee. Ich wollte das auch schon gerade machen, aber 
ich habe Linux neu installiert und FEMM noch nicht wieder drauf.

Aber ich wie bereits schrieb, reicht es nicht aus, das Magnetfeld und 
die Kraft nur zu einem Zeitpunkt zu betrachten. Relevant ist beides 
während des ganzen Vorgangs.

von Steven (muellermilch)


Lesenswert?

Florian schrieb:
> Das könnte der Fehler liegen. So sieht das Magnetfeld nicht aus.
> Wolfgang hat den richtigen Verlauf gezeigt.

Man kann auch übertreiben. Deswegen schrieb ich ja grob und nicht genau.
Das soll doch nur die Wirkung auf die Kugel darstellen.

Florian schrieb:
> Ich wollte das auch schon gerade machen

Wie würdest du es machen? Bist du Experte in FEMM?

Florian schrieb:
> Aber ich wie bereits schrieb, reicht es nicht aus, das Magnetfeld und
> die Kraft nur zu einem Zeitpunkt zu betrachten. Relevant ist beides
> während des ganzen Vorgangs.

Das ist mir natürlich bewusst.

von Alexander (alecxs)


Angehängte Dateien:

Lesenswert?

Wolfgang R. schrieb:
> Ein völlig übliches Vorgehen, um rotierende Systeme anschaulich zu
> erklären.

Ach so so, du findest also das ist ein adäquates Modell? Schau Dir 
einfach mal das Verhältnis der bewegten zur unbewegten Masse an. Niemand 
redet hier von Perpetuum Mobile, hast Du das immer noch nicht kapiert?

Das "Veranschauungsmodell" ist hat das Verhältnis völlig umgekehrt, hier 
hat nur noch die Kugel kinetische Energie, das sind 90% weniger Masse!

Wolfgang R. schrieb:
> Klar. Dann bin ich ja gespannt, wann mir einer der erleuchteten
> Psiram-Leugner den ersten funktionsfähigen Magnetmotor präsentiert.

Es gibt welche mit Nockenwelle und Federspanner die den Statormagnet an 
der 360 Grad Übergangsstelle kurz auskoppeln, und welche mit 
Massenschwungrad die durch ihr Trägheitsmoment den Übergang meistern. Es 
gibt einige die beschleunigen sogar. Hast Du noch keinen gesehen oder 
was?

Was sind Psiram-Leugner, meinst Du ANTIPSIRAM?

: Bearbeitet durch User
von Florian (flori_n)


Angehängte Dateien:

Lesenswert?

Steven schrieb:
> Florian schrieb:
>> Das könnte der Fehler liegen. So sieht das Magnetfeld nicht aus.
>> Wolfgang hat den richtigen Verlauf gezeigt.
>
> Man kann auch übertreiben. Deswegen schrieb ich ja grob und nicht genau.
> Das soll doch nur die Wirkung auf die Kugel darstellen.
Das wirkt jetzt aber schon ein bisschen trollig. Was du gezeichnet hast 
sieht vollkommen anders aus als der echte Verlauf. Das ist nichtmal grob 
richtig.

Steven schrieb:
> Florian schrieb:
>> Ich wollte das auch schon gerade machen
>
> Wie würdest du es machen? Bist du Experte in FEMM?
Nein. Ich kenne mich nur grundlegend aus.
Angehängt ist die Simulation aus FEMM. Außen Luft, die Kugel aus reinem 
Eisen, der Magnet aus SmCo24 und die Ebene aus Aluminium.

von Wolfgang R. (Firma: www.wolfgangrobel.de) (mikemcbike)


Lesenswert?

Alexander schrieb:
> Hast Du noch keinen gesehen oder
> was?

Nein, hab ich nicht, nur Fake-Krempel.
Hast du einen daheim?

von Alexander (alecxs)


Lesenswert?

Nein. Magneten sind teuer.

von Wolfgang R. (Firma: www.wolfgangrobel.de) (mikemcbike)


Lesenswert?

Alexander schrieb:
> Nein. Magneten sind teuer.

Du bist ein Troll. EOT.

von Steven (muellermilch)


Lesenswert?

Florian schrieb:
> Das wirkt jetzt aber schon ein bisschen trollig. Was du gezeichnet hast
> sieht vollkommen anders aus als der echte Verlauf. Das ist nichtmal grob
> richtig.

Du hast recht. Mit den Bildern hast du mich überzeugt, dass meine 
Zeichnung falsch ist.
Ich wollte es mir gerade in FEMM anschauen, aber du warst schneller. 
Danke für die Arbeit.
Nun erkenne ich auch die ziehende Wirkung auf die Kugel beim starren 
Magneten. Allerdings erkenne ich auch, dass der bewegte Magnet mehr 
Kraft auf die Kugel wirkt. Interessant wäre sich die Kraft punktuell 
anzuschauen, je nach Richtung. Ich weiß dass ich das mit der Flussdichte 
machen kann, aber mit der Kraft wüsste ich nicht wie.

Bitte melde dich an um einen Beitrag zu schreiben. Anmeldung ist kostenlos und dauert nur eine Minute.
Bestehender Account
Schon ein Account bei Google/GoogleMail? Keine Anmeldung erforderlich!
Mit Google-Account einloggen
Noch kein Account? Hier anmelden.